近三年MBA联考真题1

上传人:新** 文档编号:577796968 上传时间:2024-08-22 格式:PDF 页数:84 大小:16.39MB
返回 下载 相关 举报
近三年MBA联考真题1_第1页
第1页 / 共84页
近三年MBA联考真题1_第2页
第2页 / 共84页
近三年MBA联考真题1_第3页
第3页 / 共84页
近三年MBA联考真题1_第4页
第4页 / 共84页
近三年MBA联考真题1_第5页
第5页 / 共84页
点击查看更多>>
资源描述

《近三年MBA联考真题1》由会员分享,可在线阅读,更多相关《近三年MBA联考真题1(84页珍藏版)》请在金锄头文库上搜索。

1、近三年MBA联考真题目录( 一)2012年MBA综合真题及答案. . . . . . . . . . . . . . . . . . . . . . . . . .1( )2012年MBA英语真题及答案. . . . . . . . . . . . . . . . . . . . . . . . .23( 三)2011年MBA综合真题及答案. . . . . . . . . . . . . . . . . . . . . . . . .33( 四)2011年MBA英语真题及答案. . . . . . . . . . . . . . . . . . . . . . . . .47( 五)2010

2、年MBA综合真题及答案. . . . . . . . . . . . . . . . . . . . . . . . .63( 六)2010年MBA英语真题及答案. . . . . . . . . . . . . . . . . . . . . . . . .74( 一)2012年MBA综合真题及答案一、问题求解题:第 115 小题,每小题三分,共 4 5 分。下列每题给出的A 、B 、C 、D 、E 五个选项中,只有一项是符合试题要求的。请在答题卡上将所选的字母涂黑。1 . 某商品的定价为200元,受金融危机的影响,连续两次降价20%后的售价为( A ) 114 元 (B) 120 元( C

3、) 128 元 (D) 144 元(E) 160元2. 如图1, AABC是直角三角形, 果 晶 与 为 正 方形,已知, a, b, c , 分 别 是 周 易 科 的边长,则( A ) a=b+c ( B) a2=b2+c3( C) a2=2b2+2c3 ( D) a b + c ( E) a5=2b3+2c3c广州华章MBA 图 23 .如图2, 个储物罐的下半部分是底面直径与高均是20m的圆柱形、上半部分( 顶部)是半球形,已知底面与顶部的造价是400元/m,侧面的造价是300元/m:该储物罐的造价是。(M3.14)(A) 56.52 万元(C) 75.36 万元(E) 100.48

4、万元(B) 62.8 万元(D) 87.92 万元4 . 在一次商品促销活动中,主持人出示个9 位数,让顾客猜测商品的价格,商品的价格是该9 位数中从左到右相邻的3 个数字组成的3 位数,若主持人出示的是513535319,则顾客一次猜中价格的概率是11(A) 7 (B) 612(C) 5 (D) 71(E) 35 .某商店经营15种商品,每次在橱窗内陈列5 种,若每两次陈列的商品不完全相同,则最多可陈列( A ) 3000 次 (B) 3003 次( 0 4000 次 (D) 4003 次( E) 4300 次6. 甲 、乙、丙三个地区的公务员参加一次测评,其人数和考分情况如卜表:人 数 分

5、 数 地 区6789甲10101010乙15151020丙10101515三个地区按平均分山高到低的排名顺序为( A ) 乙、丙、甲 (B ) 乙、甲、丙( C ) 甲、丙、乙 (D ) 丙、甲、乙(E ) 丙、乙、甲7. 经统计,某机场的一个安检口每天中午办理安检手续的乘客人数及相应的概率如下表:乘客人数05610117516-2021-252 5 以上概率0.10.20.20.250.20.05该安检口 2 天中至少有1 天中午办理安检手续的乘客人数超过15的概率是(A)0.2 (B)0.25(C)0.4 (D)0.5(E)0.752 18 . 某人在保险柜中存放了 M 元现金,第一天取出

6、它的以后每天取出前一天所取的共取了 7 次,保险柜中剩余的现金为:M M(A) 37 元 (B) 1元2M 1(C) 3f 元 (D) 1-(孑)M 元2(E) “7 X (3)M 元9. 在直角坐标系中,若平面区域D 中所有点的坐标( 耳)均满足:。4工46,上一川M3, 一 1 八 ,则 D 的面积是; Q+旬 7)(A) 4 (B) 4(C) 4 (D) 49如(E) 410. 某单位春季植树100颗,前 2 天安排乙组植树,其余任务由甲、乙两组用3 天完成,已知甲组每天比乙组多植树4 棵,则甲组每天植树(A)11 棵 (B)12 棵(C) 13棵 (D )15棵(E) 17棵11、在两

7、队进行的羽毛球对抗赛中,每队派出3 男 2 女共5 名运动员进行5 局单打比赛。如果女子比赛安排在第二和第四局进行,则每队队员的不同出场顺序有( A ) 1 2 种 (B) 1 。种( C) 8 种 ( D) 6 种12、若能被整除,则( A) .=(占 =4 ( B)a = - 4, b 4-( C) O=10LA = T (D)。= -(E) = -X =。1 3 .某公司计划运送180台电视机和110台洗衣机下乡,现在两种货车,甲种货车每辆最多可载4 0 台电视机和1 0 台洗衣机,乙种货车每辆最多可载20台电视机和2 0 台洗衣机,已知甲、乙种货车的租金分别是每辆400元和360元,则

8、最少的运费是( A ) 2560 元 (B) 2600 元( C) 2640 元 (D) 2580 元( E) 2720元1 4 .如图3 , 三个边长为一的正方形所覆盖区域( 实线所围)的面积为(A) 3 - 石(C) 3 - 后3- 空3 - %(B) 4,再3 -(D) 2(E)15 . 在一次捐赠活动中,某市将捐赠的物品打包成件,其中帐篷和食品共320件,帐篷比食品多80件,则帐篷的件数是( A) 180 ( B) 200( C) 220 ( D) 240( E) 260二、条件充分性判断:第 1625小题,每小题3 分,共 30分。要求判断每题给出的条件( 1)和(2)能否充分支持题

9、干所陈述的结论。A、B、C、D、E 五个选项为判断结果,请选择一项符合试题要求的判断,在答题卡上将所选项的字母涂黑.( A) 条 件(1)充分,但 条 件(2)不充分。( B) 条 件(2)充分,但 条 件(1)不充分。( C) 条 件(1)和 条 件(2)单独都不充分,但 条 件(1)和 条 件(2)联合起来充分。( D) 条 件(1)充分,条 件(2)也充分。( E) 条 件(1)和 条 件(2)单独都不充分,条 件(1)和 条 件(2)联合起来也不充分。16 . 一元二次方程7 + 辰 + 1 = 0 有两个不同实根.( 1) b *22 . 在某次考试中,3道题中答对2 道题即为及格.

10、 假设某人答对各题的概率相同, 则此人及格20的概率是27.2(1)答对各题的概率均为了1(2)3道题全部答错的概率为2723 .已知三种水果的平均价格为10元/ 千克,则每种水果的价格均不超过18元/ 千克。(1)三种水果中价格最低的为6 元/ 千克。(2 ) 购买重量分别是1 千克、1 千克和2 千克的三种水果共用了 46元。24 . 某户要建一个长方形的羊栏,则羊栏的面积大于3Z.(1 )羊栏的周长为120m.(2 )羊栏对角线的长不超过50m.25 . 直线/野 = d , 曲切线.甫且仅有一个交点.(2) / - * 2- 工不数学答案: 1-5 CACBB 6-10 EEACD 1

11、1-15 ADBEB16-20 DCABD 21-25 EDDCA答案解析:一、答 案 是a解析:20 0 * 0 . 82= 128 ( 元 ).二、答 案 是A ,三、答 案 是a解析 : 。2 + 2 舛4。 。+ 2 1。*2。 卜3。 。=75 . 3 6舄黜黑四、答案是B解析:总可能性513、135、353、535、531、319共六种从而所求概率为不五、答案是B解析:CA” = 5110! = 3003六、答案是E60 + 70 + 80 + 90 7 ,解析:甲平均= 4 090 + 105 + 80 + 180 _ 455 _乙平均= 60 6060 + 70 + 120 +

12、 135 -= 7.7丙平均= 50七、答案是E解析:两天都没超过15的概率为(泥海 从而至少一天超过15人的概率为 40.75八、答案是A九、答案是C解析:36-*32-2 * - 9 = 2 7 - = 9(3 - - |424 4J卜 、答案是D解析:甲每天X颗 . 乙 每 天Y颗,5j+3.x=100产=) + 4 解得 y = 11,x=15十一、答案是A解析:2*3!=12 ( 种)十二、答案是D解析:x + x2 + a x + b = ( X-r-lX-V-2)。+6+2=0徨 2 a+6+12 = 0 即a = TO,6=8 卜三、答案是B解析:用穷举法甲2 .乙5 ,所用费

13、用为2600元最小.卜四、答案是E3 - 2 / *怎3 = 3 _迈解析: 2 2 4 4 五、答案是B工 +二320解析:卜 = ) , +8 0解得x = 200 ( 件)十六、答案是D解析:题干要求即 4,=网 2 即, 2 或 6-2十七、答案是C解析:联 合( 1)、( 2).,g + 8 / + 1 + 1 + .T1 + d = -= -qq卜八、答案是A即( 1)充分由条件( 2) ) = x T汨7成立1 / . /即条件( 2)不充分卜九、答案是B解析:由( 1)( 0-8炉0.8即( 2)充分二十、答案是D解析:由( 1) 3 , 是偶数.由( 2) 3一是偶数二十一、

14、答案为E解析:取。= -2*=-1, 从而m一定是偶数,从而m一定是偶数则知应为E二十二、答案是D解析:C; p2(l-p) + c; p3(_p = 3p2(_p)+p3_ 2 2 0由( I ) 一 得 结 果 为2 7C j( l- p = P = 由( 2 ) 2 7 ,则 3与( 1 )相同二十三、答案是D解析:由题干,设价格每千克分别为x j ,二,则x +) , +二=3 0由 若最低为6,则另两和为2 4 ,即每种价格不超过1 8 .由( 2 ) x + 产二 + 二= 46,得二=1 6从而x +) , = 14,即(2)也充分二十四、答案是C解析:若。=5 0 , 6 =1

15、 0则,知( 1 )不充分,显 然( 2 )也不充分 联 合( 1 ) ( 2 ) ,aba + 6 = 6 0 J。 ? + b2 5 0& jl( a + b)2 =a2+ b2 + lab = 3 6 0 0, a2 + tr 1 1 0 0 , a / 5 5 0 成立二十五、答案是A解析:( 1 ) = x + 6与) =一 + ”有且仅有一个交点,由于) = x + “斜率为1 ,即) = x + b不能与) 轴平行从而,只能与尸犬 + ”相切 (2) x2+ax + b ,如图从 而(2 ) 不充分.三、逻辑推理:本大题共30小题,每小题2 分, 共 60分。从下面每题所给出的五

16、个选项中,只有一项是符合试题要求的。请在答题卡上所选项的字母涂黑。26 . 1991年 6 月 1 5 日,菲律宾吕宋岛上的皮纳图博火山突然大喷发,2000万吨二氧化硫气体冲入平流层, 形成的霾像毯子一样盖在地球上空, 把部分要照射到地球的阳光反射回太空。几年之后, 气象学家发现这层使得当时地球表面的温度累计下降了 0.5.而皮纳图博火山喷发前的一个世纪,因人类活动而造成的温度效应已经使地球表面温度升高1。某位持“ 人工气候改造论” 的科学家据此认为,可以用火箭弹等方式将二氧化硫充入大气层,阻挡部分阳光,达到地球表面降温的目的。以下哪项如果为真,最能对该科学家提议的有效性构成质疑?(A)如果利

17、用火箭弹将二氧化硫充入大气层,会导致航空乘客呼吸不适。(B)如果在大气层上空放置反光物,就可以避免地球表面强烈阳光的照射。(C)可以把大气中的碳取出来存储到地下,减少大气层的碳含量。(D)不论何种方式,“ 人工气候改造” 都将破坏地球的大气层结构。(E)火山喷发形成的降温效应只是暂时的,经过一段时间温度将再次回升。答案:D题干为措施-目的型论证。关键词:“ 提议” 。措施为火箭弹等方式将二氧化硫充入大气层,阻挡部分阳光。目的是达到地球表面降温的目的。削弱的方式为措施不可行、达不到目的及措施无意义( 产生严正: 的否定性后果) 。迷惑选项通常为措施某一点不合理、有一定的副作用或其他措施在某方面更

18、好。D 项说明措施产生严重的否定性后果,措施无意义。削弱了题干论证。而A 项只是说明措施的一般性副作用。其他项和该措施无关。27 . 近期流感肆虐,般流感患者可采用抗病毒药物的治疗, 虽然并不是所有流感患者均需接受达菲等抗病毒药物的治疗, 但不少医生仍强烈建议老人、 儿童等易出现严重症状的患者用药。如果以上陈述为真, 则以哪项一定为假?I.有些流感患者需接受抗病毒药物的治疗。II . 并非有的流感患者不需接受抗毒药物的治疗。III . 老人、儿童等易出现严重症状的患者不需要用药。(A)仅 I (B )仅 II (C )仅 III (D )仅 l、ll (E )仅 II、III答案:B根据题干中

19、“ 并不是所有流感患者均需接受达菲等抗病毒药物的治疗” 可以推出(1)“ 有的流感患者不需接受达菲等抗病毒药物的治疗” ,(1)与 I 是下反对关系,因此I 不能确定真假。II为(1 ) 的矛盾命题,所以II为假。“ 不少医生仍强烈建议老人、儿童等易出现严重症状的患者用药” 并不能推出这些人是否真的需要用药。所 以 III不能确定真假。28 . 某公司规定, 在一个月内, 除非每个工作日都出勤, 否则任何员工都不可能既获得当月绩效工资, 又获得奖励工资。以下哪项与上述规定的意思最为接近?(A )在一个月内, 任何员工如果所有工作I I不出勤, 必然既获得当月绩效工资, 又获得奖励工资。(B )

20、在一个月内, 任何员工如果所有工作日不缺勤, 都有可能既获得当月绩效工资, 又获得奖励工资。(C )在在一个月内, 任何员工如果有某个工作日缺勤, 仍有可能获得当月绩效工资, 或者获得奖励工资。(D )在一个月内, 任何员工如果有某个工作日缺勤, 必然或者得不了当月绩效工资, 或者得不了奖励工资。(E )在个月内, 任何员工如果有工作日缺勤, 必然既得不了当月绩效工资, 又得不了奖励工资。答案:D按上课所教“ 除非P 否则Q”转化为“ 如果非P,则 Q”的方式将题干中命题转变为:“ 如果有某个工作日不出勤,则任何员工都不可能既获得当月绩效工资, 又获得奖励工资。”而“ 不可能既获得当月绩效工资

21、, 又获得奖励工资” 等价于“ 必然没获得当月绩效工资或没获得奖励工资” 。 ( 不可能P 且 Q=必然非P 或非Q)所以选D.29 . 只有具有一定文学造诣且具有生物学专业背景的人,才能读懂这篇文章。如果上述命题为真,以下哪项不可能为真?(A)小张没有读懂这篇文章,他但的文学造诣是大家所公认的。(B)计算机专业的小王没有读懂这篇文章。(C)从未接触过生物学知识的小李读懂了这篇文章。(D)小周具有生物学专业背景,但他没有读懂这篇文章。(E)生物学博士小赵读懂了这篇文章。答案:C本题考点为必要条件的负命题。“ 只有P 才 Q”为真,则其负命题“ 非 P 且 Q”不可能真。题干为真,则“ 没有一定

22、文学造诣或没有生物学专业背景,但能读懂这篇文章” 不可能真。所以C 不可能为真。30 . 经过反复核查,质检员小李向厂长汇报说:“726车间生产的产品都是合格的,所以不合格的产品都不是726车间生产的。”以下哪项和小李的推理结构最为相似?(A)所有入场的考生都经过了体温测试,所以没有入场的考生都没有经过体温测试。(B 所有出厂设备都是检测合格的,所以检测合格的设备都已出厂。(C)所有已发表文章都是认真校对过的,所以认真校对过的文章都已发表。(D)所有真理都是不怕批评的,所以怕批评的都不是真理。(E)所有不及格的学生都没有好好复习,所以没好好复习的学生都不及格。答案:D题干推理结构为:“ 所有S

23、 是 P , 所以, 所有不是P 的都不是S”。D 项与题 相同。31 . 王涛和周波是理科(1 ) 班同学,他们是无话不说的好朋友。他们发现班里每一个人或者喜欢物理,或者喜欢化学。王涛喜欢物理,周波不喜欢化学。根据以上陈述,以下哪项必定为真?I . 周波喜欢物理。II . 王涛不喜欢化学。川. 理科(1 ) 班不喜欢物理的人喜欢化学。IV.理 科(1 ) 班一半喜欢物理,一半喜欢化学。(A)仅 I (B )仅 III (C )仅 I、II (D )仅 I、III (E )仅 II、川、IV答案:D本题考核不相容选言P 或 Q 的推理( 否定一支推出肯定另一支) 。所以周波不喜欢化学推出周波喜

24、欢物理。同理,理 科(1)班不喜欢物理的推出喜欢化学。因此I 和 川 正确。其它无法推出。所以选D。32 . 李明、王兵、马云三位股民对股票A 和股票B 分别做了如下预测:李明:只有股票A 不上涨,股票B 才不上涨。王兵:股票A 和股票B 至少有一个不上涨。马云:股票A 上涨当且仅当股票B 上涨。若三人的预测都为真,则以下哪项符合他们的预测?(A )股票A 上涨,股票B 才不上涨。(B )股票A 不上涨,股票B 上涨。(C )股票A 和股票B 均上涨。(D )股票A 和股票B 均不上涨。(E )只有股票A 上涨,股票B 才不上涨。答案:D根据马云为真,推出股票A 是股票B 上涨的充要条件,即二

25、者同时上涨或同时不上涨。再根据王兵为真,可知两者同时不上涨。33 . 临江后地处东部沿海,下辖临东、临西、江南、江北四个区,近年来,文化旅游产业成为该市的经济增长点。 2010年, 该市一共吸引全国数卜万人次游客前来参观旅游。 12月底,关于该市四个区吸引游客人数多少的排名,各位旅游局长作了如下预测:临东区旅游局长:如果临西区第三,那么江北区第四;临西区旅游局长:江南区旅游局长:江北区旅游局长:最终的统计表明,只有临西区不是第一,江南区才是第二;江南区不是第二;江北区第四。只有一位局长的预测符合事实,则临东区当年吸引游客人次的排名是(A)第一 (B)第 二(C)第 三(D)第 四(E)在江北区

26、之前答案:D本题为真假话推理。题干中没有矛盾及下反对命题,采用假设法。题干条件调整为:(1)临西第三则江北第四( 2 )临西第则江南不是第二( 3 )江南不是第二( 4 )江北第四假 设(1)为假,推出临西第三且江北第四假 设(2 )为假,推出临西第一且江南第二所 以 当(1) (2)同时为假时推出临西第一且笫三,明显不可能。所 以(1) (2)至少一个为真,即二者一真一假,可 知(5 )临西第一或第三。再山题干只有一真,可 知(3) ( 4 )为假。所以江南第二、江北不是第四,所 以(6 )江北是第 或第三。所 以 山(5) ( 6 )可知临西和江北分别是第和第三。而江南第二,那么临东就是第

27、四。34 . 小张是某公司营销部的员工。公司经理对他说:“ 如果你争取到这个项目,我就奖励你一台笔记本电脑或者给你项目提成。”以下哪项如果为真,说明该经理没有兑现承诺?(A)小张没有争取到这个项目,该经理没有给他项目提成,但送了他一台笔记本电脑。(B)小张没有争取到这个项目,该经理没奖励给他笔记本电脑,也没给他他项目提成。(C )小张争取到了这个项目,该经理给他项目提成,但并未奖励他笔记本电脑。(D )小张争取到了这个项目,该经理奖励他一台笔记本电脑并给他三天假期。(E )小张争取到了这个项目,该经理未给他项目提成,但 奖 励 了 他 哈 台 式 电 脑 。答案:E本题考核充分条件假言负命题。

28、即“ 如 果P则Q”的负命题为“P且非Q”。所 以E为真,即“P且 非Q”为真时,说明 如果P则Q”不成立。35 . 文化新报记者小白周四去某市采访陈教授与王研究员。次日,其同事小李问小白:“ 昨天你采访到那两位学者了吗? ” 小白说:“ 不,没那么顺利。” 小李又问:“ 那么,你一个都没采访到? ” 小 白说:“ 也不是。”以下哪项最有可能是小白周四采访所发生的真实情况?(A)小白采访到了两位学者。(B)小白采访了李教授,但没有采访王研究员。(C)小白根本没有去采访两位学者。(D)两位采访对象都没有接受采访。(E)小白采访到了其中一位,但是没有采访到另一位。答案:E题 中小白对“ 两位都采访

29、到” 和“ 两位都没采访到” 都进行了否定,说明“ 有一位采访到,而另位没采访到” 。36 . 只有通过身份认证的人才允许上公司内网,如果没有良好的业绩就不可能通过身份认证,张辉有良好的业绩而王纬没有良好的业绩。如果上述断定为真,则以下哪项一定为真?(A)允许张辉上公司内网。(B)不允许王纬上公司内网。(C)张辉通过身份认证。(D)有良好的业绩,就允许上公司内网。(E)没有通过身份认证,就说明没有良好的业绩.答案:B本题考核假言命题推理规则,即充分条件推理规则为肯前推出肯后,否后推出否前;必要条件推理规则为肯后推出肯前,否前推出否后。题干条件可转化为:没有良好业绩=不可能通过身份认证=不允许上

30、公司内网再根据王纬没有良好的业绩,推出不允许王纬上公司内网。所以B 正确。37 . 比较文字学者张教授认为,在不同的民族语言中,字形与字义的关系有不同的表现。他提出,汉字是象形文字,其中大部分的是形声字,这些字的字形与字义相互关联;而英语是拼音文字,其字形与字义往往关联度不大,需要某种抽象的理解。以下哪像如果为真,最不符合张教授的观点?(A )汉语中的“ 日” 、 “ 月” 是象形字, 从字形可以看出其所指的对象; 而英语中的sun与 moon则感觉不到这种形义结合。(B ) 汉语中的“ 日” 与 沐 ” 结合,可以组成 柬 、 杲 、“ 杳” 等不同的字,并可以猜测其语义。而英语中则不存在与

31、此类似的sun与 wood的结合。(C ) 英语中,也有与汉语类似的象形文字,如,eye是人的眼睛的象形,两个e 代表眼睛,y 代表中间的鼻子;bed是床的象形,b 和 d 代表床的两端。(D ) 英语中的sunlight与汉语中的“ 阳光” 相对应, 而英语的sun与light和汉语中的“ 阳” 与“ 光”相对应。( E ) 汉语的“ 星期三” 与英语中的Wednesday和德语中的Mitwoch意思相同。答案:CC 项直接说明英语中也有象形文字,直接否定张教授的观点。38 . 乘客使用手机及便携式电子设备会通过电磁波谱频繁传输信号, 机场的无线电话和导航网络等也会使用电磁波谱,但也信委员会

32、已根据不同用途把电磁波谱分成了几大块。因此,用手机打电话不会对专供飞机通讯系统或全球定位系统使用的波段造成干扰。 尽管如此, 各大航空公司仍然规定,禁止机上乘客使用手机等电子设备。以下哪项如果为真,能解释上述现象?I . 乘客在空中使用手机等电子设备可能对地面导航网络造成干扰。H.乘客在起飞和降落时使用手机等电子设备,可能影响机组人员工作。in.便携式电脑或者游戏设备可能导致自动驾驶仪出现断路或仪器显示发生故障。( A ) 仅 I ( B ) 仅 n ( C ) 仅 1、IJ(D)仅n 、in (E)i 、n 和in答案:EI、ii和川三项均说明了禁止机上乘客使用手机等电子设备的原因,所以均能

33、解释。39 . 2010年上海世博会盛况空前,200多个国家场馆和企业主题馆让人目不暇接。大学生王刚决定在学校放暑假的第二天前往世博会参观。 前一天晚上, 他特别上网查看了各位网友相对热门场馆选择的建议,其中最吸引王刚的有三条:( 1)如果参观沙特馆,就不参观石油馆。( 2 )石油馆和中国国家馆择一参观。(3)中国国家馆和石油馆不都参观。实际匕第二天王刚的世博会行程非常紧凑,他没有接受上述三条建议中的任何一条。关于王刚所参观的热门场馆,以下哪项描述正确?(A )参观沙特馆、石油馆,没有参观中国国家馆。(B)沙特馆、石油馆、中国国家馆都参观了。(C)沙特馆、石油馆、中国国家馆都没有参观。(D)没

34、有参观沙特馆,参观石油馆和中国国家馆。(E)没有参观石油馆,参加沙特馆、中国国家馆。答案:B根 据 ( 1)假可知参观了沙特馆且参观了石油馆。再 根 据(3)为假推出中国国家馆和石油馆都参观了。因此王刚参观了沙特馆、石油馆以及中国国家馆。所以BiE确。40 . 经理说:“ 有了自信不一定赢” 。董事长回应说:“ 但是没有自信一定会输。”以下哪项与董事长的意思最为接近?(A )不输即赢,不赢即输。(B)如果自信,则一定会赢。(C)只有自信,才可能不输。(D)除非自信,否则不可能输。(E)只有赢了,才可能更自信。答案:C董事长意思为:如果没有自信则一定会输。C 项只有自信,才可能不输。根据必要条件

35、命题推理规则,否定前件推出否定后件。所以,如果不自信,则不可能不输;而不可能不输等价于一定会输,所以根据C 项与董事长意思最为接近。41 . 在家电产品“ 三下乡” 活动中,某销售公司的产品受到了农村居民的广泛欢迎。该公司总经理在介绍经验时表示: 只有用最流行畅销的明星产品面对农村居民, 才能获得他们的青睐。以下哪项如果为真,最能质疑总经理的论述?( A) 某品牌电视由于其较强的防潮能力,尽管不是明星产品,仍然获得了农村居民的青睐。( B) 流行畅销的明星产品由于价格偏高,没有赢得农村居民的青睐。( C) 流行畅销的明星产品只有质量过硬,才能获得农村居民的青睐。( D) 有少数娱乐明星为某些流

36、行畅销的产品作虚假广告。( E) 流行畅销的明星产品最适合城市中的白领使用。答案:A必要条件“ 只有P 才 Q”的负命题为“ 非 P 且 Q”。所以,“ 非 P 且 Q”成立,说明 只有P 才 Q”不成立。A 项正好采用此方式否定了总经理所说的必要条件命题。42 . 居民苏女士在菜市场看到某摊位出售的鹤鹑蛋色泽新鲜、形态圆润,且价格便宜,于是买了一箱。回家后发现有些鹤鹑蛋打不破,甚至丢在地上也摔不坏,再细闻已经打破的鹤鹑蛋,有一股刺鼻的消毒液味道。她投诉至菜市场管理部门,结果一位工作人员声称鹤鹑蛋目前还没有国家质量标准,无法判定它有质量问题,所以他坚持这箱鹤鹑蛋没有质量问题。以下哪项与该工作人

37、员作出结论的方式最为相似?(A)不能证明宇宙是没有边际的,所以宇宙是有边际的。(B)“ 驴友论坛” 还没有论坛规范,所以管理人员没有权利删除贴子。(C)小偷在逃跑途中跳入2 米深的河中,事主认为没有责任,因此不予施救。(D)并非外星人不存在,所以外星人存在。(E)慈善晚会上的假唱行为不属于商业管理范围,因此相关部门无法对此进行处罚。答案:A题 根据“ 无法判定它有质量问题推出没有质量问题” 是一种诉诸无知的推理(P 并被不知道或没被证明为真,所以P 是 假 的 ) 。例如, 我们上课中常讲的“ 我们不知道鬼存在,所以鬼不存在。” “ 没有外星人与人类交往的记录,所以外星人没有访问地球。”A 项

38、也是诉诸无知的推理。43 . 概念A 与概念B 之间有交叉关系,当且仅当, (1 ) 存在对象X, x 既属于A 又属于B;(2 ) 存在对象y, y 属于A 但不属于B; ( 3 ) 存在对象z, z 属于B 但是不属于A 。根据上述定义,以下哪项中加点的两个概念之间有交叉关系?(A ) 国画按题材分主要有物画、花鸟画、山水画等等;按技法分主要有工笔画和写意画等等。(B) 盗梦空间除了是最佳影片的有力争夺者外, 它在技术类奖项的争夺中也将有所斩获。(C ) 洛邑小学30岁的食堂总经理为了改善伙食, 在食堂放了儿个意见本,征求学生们的意见。(D ) 在微波炉清洁剂中加入漂白剂, 就会释放出氯气

39、。(E ) 高校教师包括教授、副教授、讲师和助教等。答案:A本题考察概念之间的交叉关系,较为简单。44 . 小李将自家护栏边的绿地毁坏,种上了黄瓜。小区物业人员发现后,提醒小李:护栏边的绿地是公共绿地,属于小区的所有人。物业为此下发了整改通知书, 要求小李限期恢复绿地。小李对此辩称:“ 我难道不是小区的人吗?护栏边的绿地既然属于小区的所有人,当然也属于我。因此,我有权在自己的土地上种瓜。”以下哪项论证,和小李的错误最为相似?(A )所有人都要为他的错误行为负责,小梁没有对他的错误行为负责,所以小梁的这次行为没有错误。(B )所有参展的兰花在这次博览会上被定购一空,李阳花大价钱买了一盆花,山此可

40、见,李阳买的必定是兰花。(C )没有人能够 天读完大仲马的所有作品,没有人能够一天读完 三个火枪手,因此, 三个火枪手是大仲马的作品之一。(D )所有莫尔碧骑士组成的军队在当时的欧洲是不可战胜的,翼雅王是莫尔碧骑士之一,所以翼雅王在当时的欧洲是不可战胜的。(E )任何一个人都不可能掌握当今世界的所有知识,地心说不是当今世界的知识,因此,有些人可以掌握地心说。答案:D小李的推理为:护栏边的绿地既然属于小区的所有人,我是小区的人,所以护栏边的绿地也属于我。此推理中的“ 所有人” 在句中是一个集合概念,并非指“ 每一个人” ,这是典型的四概念错误。D形式及错误与题干相似。45 .我国著名的地质学家李

41、四光, 在对东北的地质结构进行了长期、 深入的调查研究后发现,松辽平原的地质结构与中亚细亚极其相似。他推断,既然中亚细亚蕴藏大量的石油,那么松辽平原很可能也蕴藏的大量的石油。后来,大庆油田的开发证明了李四光的推断是正确的。以下哪项与李四光的推理方式最为相似?(A )他山之石,可以攻玉。(B )邻居买彩票中了大奖,小张受此启发,也去买了体育彩票,结果没有中奖。(C )某乡镇领导在考察了荷兰等地的花卉市场后认为要大力发展规模经济, 回来后组织全乡镇种大葱,结果导致大葱严重滞销。(D )每到炎热的夏季,许多商店腾出大块地方卖羊毛衫、长袖衬衣、冬靴等冬令商品,进行反季节销售,结果都很有市场。小王受此启

42、发,决定在冬季种植西瓜。(E)乌兹别克地区盛产长绒棉。新疆塔里木河流域与乌兹别克地区在日照情况、霜期长短、气温高低、降雨量等方面均相似,科研人员受此启发,将长绒棉移植到塔里木河流域,果然获得了成功。答案:E题 干 推 理 为 类 比 推 理 根 据A和B在某些方面相同或相似推出在其他方面也相同或相似。E明显与题干最为相似。46 .如果他勇于承担责任,那么他就一定会直面媒体,而不是选择逃避;如果他没有责任,那么他就, 定会聘请律师,捍卫自己的尊严。可是事实上,他不仅没有聘请律师,现在逃得连人影都不见了。根据以上陈述,可以得出以下哪项结论?(A )即使他没有责任,也不应该选择逃避。(B)虽然选择了

43、逃避,但是他可能没有责任。(C)如果他有责任,那么他应该勇于承担责任。(D)如果他不敢承担责任,那么说明他责任很大。(E)他不仅有责任,而且他没有勇气承担责任。答案:E题干条件转化为:(1 )勇于承担责任,那么不选择逃避;(2 )没有责任,那么一定会聘请 律 师(3 )没有聘请律师,且选择了逃避。根据充分条件推理规则( 否后推出否前) ,推出他不勇于承担责任且有责任。所 以E项正确。47 .有些通讯网络维护涉及个人信息安全,因而,不是所有通信网络的维护都可以外包。以下哪项可以使上论证成立?(A)所有涉及个人信息安全的都不可以外包。(B)有些涉及个人信息安全的不可以外包。(C)有些涉及个人信息安

44、全的可以外包。(D)所有涉及国家信息安全的都不可以外包。(E)有些通讯网络维护涉及国家信息安全。答案:A题干为有些通讯网络维护涉及个人信息安全,推出有的通信网络的维护不可以外包。这是个省略的三段论。按课堂上讲的排除法结合三段论推理规则可迅速解题。首先根据两特无结排除B、 C、 E:然后根据中项“ 涉及个人信息安全” 至 少 周 延 诙 , 排 除 E。所以选A。48 . 葡萄酒中含有白藜芦醇和类黄酮等对心脏有益的抗氧化剂。一项新研究表明,白藜芦醇能防止骨质疏松和肌肉萎缩。由此,有关研究人员推断,那些长时间在国际空间站或宇宙飞船上的宇航员或许可以补充一下白藜芦醇。以下哪项如果为真,最能支持上述研

45、究人员的推断?(A )研究人员发现由于残疾或者其他因素而很少活动的人会比经常活动的人更容易出现骨质疏松和肌肉萎缩等这症状,如果能喝点葡萄酒,则可以获益。(B )研究人员模拟失重状态,对老鼠进行试验,一个对照组未接受任何特殊处理,另一组则每天服用白藜芦醇。 结果对照组的老鼠骨头和肌肉的密度都降低了, 而服用白藜芦醇的一组则没有出现这些症状。(C )研究人员发现由于残疾或者其他因素而很少活动的人,如果每天服用一定量的白藜芦醇,则可以改善骨质疏松和肌肉萎缩等症状。(D )研究人员发现,葡萄酒能对抗失重所造成的负面影响。(E )某医学博士认为,白藜芦醇或许不能代替锻炼,但它能减缓人体某些机能的退化。答

46、案:C本题的论据是:白藜芦醇能防出骨质疏松和肌肉萎缩。结论是:那些长时间在国际空间站或宇宙飞船上的宇航员或许可以补充一下白藜芦醇。这是个因推果的论证。C 项指出了因和果之间的联系,直接支持了该因果关系。本题也可以理解为措施- 目的论证。措施为补充一下白藜芦醇,目的是使那些长时间在国际空间站或宇宙K船上的宇航员防止骨质疏松和肌肉萎缩。C 项说明措施可达目的。49 . 一般商品只有在多次流通过程中才能不断增值, 但艺术品作为一种特殊商品却体现出了与一般商品不同的特征。在拍卖市场上,有些古玩、字画的成交价有很大的随机性,往往会直接受到拍卖现场气氛、 竞价激烈程度、买家心理变化等偶然因素的影响,成交价

47、有时会高于底价几卜倍乃至数百倍,使得艺术品在一次“ 流通” 中实现大幅度增值。以下哪项最无助于解释上述现象?(A )艺术品的不可再造性决定了其交换价格有可能超过其自身价值。(B )不少买家喜好收藏,抬高了艺术品的交易价格。(C )有些买家就是为了炒作艺术品,以期获得高额利润。(D )虽然大量腐品充斥市场,但是对艺术品的交易价格没有什么影响。(E )国外资金进入艺术品拍卖市场,对价格攀升起到了拉动作用。答案:D题干需要解释的现象为: 一般商品只有在多次流通过程中才能不断增值, 而艺术品却与其不同,在一次“ 流通” 中就可能实现大幅度增值。D 项明显不能解释。其它各项均从艺术品为什么价格高,大幅增

48、值的角度进行了解释。5 0 .近期国际金融危机对毕业生的就业影响非常大, 某高校就业中心的陈老师希望广大同学能够调整自己的心态和预期。 他在一次就业指导会上提到, 有些同学对自己的职业定位还不够准确。如果陈老师的陈述为真,则以下哪项不一定为真?1 . 不是所有人对自己的职业定位都准确。II . 不是所有人对自己的职业定位都不够准确。III . 有些人对自己的职业定位准确。IV . 所有人对自己的职业定位都不够准确。(A )仅 n 和W (B )仅III和w(C )仅 II 和III (D )仅 I 、 II 和HI(E )仅 II、III和IV答案:E本题为直言的判断真假题型。题干中已知条件为

49、:有些同学对自己的职业定位还不够准确。根据我总结的必记考点,已知特称为真,则其矛盾命题为假, 其他命题不能确定真假。即“ 所有人对自己的职业定位都准确” 为假,其他不确定。因此I 为真,其它均不确定。51 . 一位房地产信息员通过对某地的调查发现: 护城河两岸房屋的租金都比较廉价; 廉租房都坐落在凤凰山北麓;东向的房屋都是别墅; 非廉租房不可能具有廉价的租金; 有些单室套的两限房建在凤凰山北麓; 别墅也都建筑在凤凰山南麓。根据该房地产信息员的调查,以下哪项不可能存在?(A )东向的护城河两岸的房屋 (B )凤凰山北麓的两限房(C )单室套的廉租房 (D )护城河两岸的单室套(E )南向的廉租房

50、答案:A采用画图法解决。画出图形后,很容易看出A 不正确。52 . 探望病人通常会送上一束鲜花. 但某国曾有报道说, 医院花瓶的水可能含有很多细菌, 鲜花会在夜间与病人争夺氧气, 还可能影响病房里电子设备的工作。这引起了人们对鲜花的恐慌, 该国一些医院甚至禁止在病房内摆放鲜花。尽管后来证实鲜花并未导致更多的病人受感染, 并且权威部门也澄清, 未见任何感染病例与病房里的植物有关, 但这并未减轻医院对鲜花的反感。以下除哪项外, 都能减轻医院对鲜花的担心?(A)鲜花并不比病人身边的餐具、饮料和食物带有更多可能危害病人健康的细菌.(B)在病房里放置鲜花让病人感到心情愉悦、精神舒畅, 有助于病人康复.(

51、C)给鲜花换水、修剪需要一定的人工, 如果花瓶倒了还会导致危险产生.(D)有研究证明, 鲜花对病房空气的影响微乎其微, 可以忽略不计.( E) 探望病人所送的鲜花大都花束小、需水量少、花粉少, 不会影响电子设备工作.答案:CA、D、E 说明鲜花带来的危害很小,B 项说明鲜花有益,这些都减轻了医院对鲜花的担心。C 项明显不能减轻医院对鲜花的担心,反而有可能增加担心。53-55题基于以下题干:东宇大学公开招聘3 个教师职位,哲学学院、管理学院和经济学院各一个。每个职位都有分别来自南山大学、西京大学、北清大学的候选人。有位“ 聪明” 人士李先生对招聘结果作出了如下预测:( 1)如果哲学学院录用北清大

52、学的候选人,那么管理学院录用西京大学的候选人;(2)如果管理学院录用南山大学的候选人,那么哲学学院也录用南山大学的候选人;(3)如果经济学院录用北清大学或者西京大学的候选人,那么管理学院录用北清大学的候选人。53 . 如果哲学学院、管理学院和经济学院最终录用的候选人的大学归属信息依次如下,则哪项符合李先生的预测?(A )南山大学、南山大学、西京大学(B)北清大学、南山大学、南山大学(C)北清大学、北清大学、南山大学(D)西京大学、北清大学、南山大学(E)西京大学、西京大学、西京大学答案:D根据题干充分条件,采用排除法解题。根 据(1),可排除B、C;根 据 ( 3), 排除A 和 E;所以答案

53、选D。54 . 若哲学学院最终录用西京大学的候选人,则以下哪项表明李先生的预测错误?(A )管理学院录用北清大学候选人。(B)管理学院录用南山大学候选人。(C)经济学院录用南山大学候选人。(D)经济学院录用北清大学候选人。(E)经济学院录用西京大学候选人。答案:B管理学院录用南山大学候选人,再加上题下条件:哲学学院最终录用西京大学的候选人,说明(2 )为 假(P 且非Q 为真,说明如果P 则 Q 为假)。55 . 如果三个学院最终录用的候选人分别来自不同的大学, 则以卜硼项符合李先生的预测?(A)(B)(C)(D)(E)哲学学院录用西京大学候选人,哲学学院录用南山大学候选人,哲学学院录用北清大

54、学候选人,哲学学院录用西京大学候选人,哲学学院录用南山大学候选人,经济学院录用北清大学候选人。管理学院录用北清大学候选人。经济学院录用西京大学候选人。管理学院录用南山大学候选人。管理学院录用西京大学候选人。答案:B假设法。假设哲学院录用北清候选人,根 据(1)推出管理学院录用西京的候选人;再根据(3), 否后推出否前,推出经济学院录用南山候选人。所以排除C。假设哲学学院录用西京候选人,根据54题结果,可排除A 和 D。假设哲学学院录用南山候选人,则经济学院录用北清或西京候选人,根 据(3)推出管理学院录用北清候选人。B 项符合。四、写作:第 56-57小题,共 65分。其中论证有效性分析30分

55、,论说文35分。请写在答题卡相应位置上。56 . 论证有效性分析:分 析 F述论证中存在的缺陷和漏洞,选择若干要点,写一篇600字左右的文章,对该论证的有效性进行分析和评述。( 论证有效性分析的一般要点是:概念特别是核心概念的界定和使用是否准确并前后一致, 有无各种明显的逻辑错误, 论证的论据是否支持结论,论据成立的条件是否充分等等。)地球的气候变化已经成为当代世界关注的热点。这一问题看似复杂,其实简单。只要我们运用科学原理如爱因斯坦的相对论去对待,也许就会找到解决这一问题的方法。众所周知, 爱因斯坦提出的相对论颠覆了人类关于宇宙和自然的常识性观念。 不管是狭义相对论还是广义相对论,都揭示了宇

56、宙间事物运动中普遍存在的相对性。既然宇宙间万物的运动都是相对的, 那么我们观察问题时也应该采用相对的方法, 如变换视角等等。假如我们变换视角去看一些问题,也许会得出和一般常识完全不同的观点。 例如,我们称之为灾害的那些自然现象,包括海啸、地震、台风、暴雨等等。其实也是大自然本身的 般现象而已,从大自然的视角来看,无所谓灾害不灾害。只是当它损害了人类利益,危及了人类生存的时候,从人类的视角来看,我们才称之为灾害。假如再变换一下视角,从一个更广泛的范围来看,连我们人类自己也是大自然的一个部分。既然我们的祖先是类人猿, 而类人猿正像大熊猫、华南虎、 藏羚羊、 扬子鳄乃至银杏、 水杉、五针松等等一样,

57、是整个自然生态中的有机组成部分,那为什么我们自己就不是了呢?由此可见,人类的问题就是大自然的问题,即使人类在某时刻部分第改变了气候,也还是整个大自然系统中的一个自然问题,自然问题自然会解决,人类不必过于干涉。57 . 论说文: 根据下述材料,写一篇700字左右的论说文,题目自拟。中国现代著名哲学家熊十力先生在 十力语要( 卷一) 中说:“ 吾国学人,总好追逐风气,一时之所尚,则群起而趋其途,如海上逐臭之夫,莫名所以。曾无一刹那,风气或变,而逐臭者复如故。此等逐臭之习,有两大病。一、各人无牢固与永久不改之业,遇事无从深入,徒养成浮动性。二、大家共趋于世所矜尚之一途,则其余千途万途,一切废弃,无人

58、过问。此二大病,都是中国学人死症。”论说文参考答案熊十力先生不仅指出了我国学者做学问普遍存在的“ 跟风” 、“ 效仿” 现象,还分析了其成因及导致的后果。可以从以下角度立意:1 .深究跟风效仿的原因可形成的主题:没有恒久追求目标难成真正功业,无所自持何能恒久,不能由表及里怎能把握事物规律2 .分析跟风效仿的危害可形成的主题:浮躁是一种危害颇深的社会病,效仿是( 开拓) 创新的桎梏,千篇一律难显事物多样性3 .探讨避免跟风效仿条件可以结合实际,形成相关具体主题。( 二 )2012年MBA英语真题及答案Section I Use of EnglishDirections:Read the foll

59、owing text. Choose the best word(s) for each numbered black and mark A, B,C or D on ANSWER SHEET 1. (10 points)Millions of Americans and foreigners see G.L Joe as a mindless war toy, the symbol ofAmerican military adventurism, but thafs not how it used to be. To the men and womenwho 1 in World War I

60、I and the people they liberated, the G.l. was the 2 man growninto hero, the pool farm kid torn away from his home, the guy who 3 all the burdens ofbattle, who slept in cold foxholes, who went without the 4 of food and shelter, who stuckit out and drove back the Nazi reign of murder. This was not a v

61、olunteer soldier, notsomeone well paid, 5 an average guy, up _6_ the best trained, best equipped, fiercest,most brutal enemies seen in centuries.His name is not much.GI. is just a military abbreviation 7 Government Issue, and itwas on all of the article 8 to soldiers. And Joe? A common name for a gu

62、y whonever 9 it to the top. Joe Blow, Joe Magrac. a working class name. The United Stateshas 10 had a president or vicepresident or secretary of state Joe.G.l. joe had a 11 career fighting German .Japanese, and Korean troops. He appersas a character, or a 12 of american personalities, in the 1945 mo

63、vie The Story of G.l.Joe, based on the last days of war correspondent Ernie Pyle. Some of the soldiersPyle 13 portrayde themselves in the film. Pyle was famous for covering the 14 side ofthe warl, writing about the dirt-snow -and-mud soldiers, not how many miles were 15 orwhat towns were captured or

64、 liberated, His reports 16 the “willie“ cartoons of famedStars and Stripes artist Bill Maulden. Both men 17 the dirt and exhaustion of war,the 18 of civilization that the soldiers shared with each other and the civilians: coffee,tobacco, whiskey, shelter, sleep. 19 Egypt, France, and a dozen more co

65、untries, G.LJoe was any American soldier, 20 the most important person intheir lives.1.AperformedBservedCrebelledDbetrayed2.AactualBcommonCspecialDnormal3.AboreBcasedCremovedDloaded4.AnecessitiesBfaciliticeCJcommoditiesDpropertoes5.AandBnorCbutDhence6.AforBintoCform Dagainst7.AmeaningBimplyingCsymbo

66、lizingDclaiming8.Ahanded outBturn overCbrought backDpassed down9.ApushedBgotCmadeDmanaged1O.AeverBneverCeitherDneither11.AdisguisedBdisturbedCdisputedDdistinguished12.AcompanyBcollectionCcommunityDcolony13.AemployedBappointedCinterviewed Dquestioned14.AethicalBmilitaryCpoliticalDhuman15.AruinedBcomm

67、utedCpatrolledDgained16.AparalleledBcounteractedCduplicatedDcontradicte17.AneglectedBavoidedCemphasizedDadmired18.AstagesBillusionsCfragmentsDadvancea19.AWithBToCAmongDBeyond20.Aon the contrary B by this means Cfrom the outset Dat that pointSection II Reading ComprehensionPart ADirections:Read the f

68、ollowing four texts. Answer the questions after each text by choosing A, B, C orD. Mark your answers on ANSWER SHEET 1. (40points)Text 1Homework has never been terribly popular with students and even many parents, but inrecent years it has been particularly scorned. School districts across the count

69、ry, mostrecently Los Angeles Unified, are revising their thinking on his educational ritual.Unfortunately, L.A.Unified has produced an inflexible policy which mandates that with theexception of some advanced courses, homework may no longer count for more than 10%of a students academic grade.This rul

70、e is meant to address the difficulty that students from impoverished or chaotichomes might have in completing their homework. But the policy is unclear andcontradictory. Certainly, no homework should be assigned that students cannot do withoutexpensive equipment. But if the district is essentially g

71、iving a pass to students whodo not do their homework because of complicated family lives, it is going riskily close tothe implication that standards need to be lowered for poor children.District administrators say that homework will still be a pat of schooling: teachers areallowed to assign as much

72、of it as they want. But with homework counting for no morethan 10%of their grades, students can easily skip half their homework and see vey littledifference on their report cards. Some students might do well on state tests withoutcompleting their homework, but what about the students who performed w

73、ell on the testsand did their homework? It is quite possible that the homework helped. Yet rather thanempowering teachers to find what works best fortheir students, the policy imposes a flat,across-the-board rule.At the same time, the policy addresses none of the truly thorny questions about homewor

74、k.If the district finds homework to be unimportant to its students, academic achievement, itshould move to reduce or eliminate the assignments, not make them count for almostnothing. Conversely,if homework mattersjt should account for asignificant portion of thegrade.Meanwhile,this policy does nothi

75、ng to ensure that the homework students receive ismeaningful oraooropriate to theirage and the subject.or that teachers are not assigningmore than they are willing to review and correct.The homework rules should be put on hold while the school board, which is responsiblefor setting educational polic

76、y, looks into the matter and conducts public hearings. It is nottoo late for L.A. Unified to do homework right.21.l t is implied in paragraph 1 that nowadays h o m e w o r k .A is receiving more criticismBis no longer an educational ritualCis not required for advanced coursesDis gaining more prefere

77、nces22. L.A.Unified has made the rule about homework mainly because poor s t u d e n t s .Atend to have moderate expectations for their educationBhave asked for a different educational standardCmay have problems finishing their homeworkDhave voiced their complaints about homework23. According to Par

78、agraph 3,one problem with the policy is that it ma y .Adiscourage students from doing homeworkBresult in students indifference to their report cardsCundermine the authority of state testsDrestrict teachers* power in education24. As mentioned in Paragraph 4, a key question unanswered about homework i

79、sw h e t h e r .A it should be eliminatedBit counts much in schoolingCit places extra burdens on teachersDit is important for grades25. A suitable title for this text could be.AWrong Interpretation of an Educational PolicyBA Welcomed Policy for Poor StudentsCThorny Questions about HomeworkDA Faulty

80、Approach to HomeworkText 2Pretty in pink: adult women do not remember being so obsessed with the colour, yet itis pervasive in our young girls* lives. It is not that pink is intrinsically bad, but it is such atiny slice of the rainbow and, though it may celebrate girlhood in one way, it alsorepeated

81、ly and firmly fuses girls* identity to appearance. Then it presents that connection,even among two-year-olds, between girls as not only innocent but as evidence ofinnocence. Looking around, I despaired at the singular lack of imagination about girlslives and interests.Girls attraction to pink may se

82、em unavoidable, somehow encoded in their DNA, butaccordingto Jo Paoletti, an associate professor of American Studies, it is not. Children were not colour-coded at all until the early 20th century: in the era before domesticwashing machines all babies wore white as a practical matter, since the only

83、way ofgetting clothes clean was to boil them. Whafs more, both boys and girls wore what werethought of as gender-neutral dresses. When nursery colours were introduced, pink wasactually considered the more masculine colour, a pastel version of red, which wasassociated with strength. Blue, with its in

84、timations of the Virgin Mary, constancy andfaithfulness, symbolised femininity. It was not until the mid-1980s, when amplifyingage and sex differences became a dominant childrens marketing strategy, thatpink fully came into its own, when it began to seem inherently attractive to girls, part ofwhat d

85、efined them as female, at least for the first few critical years.I had not realised how profoundly marketing trends dictated our perception of what isnatural to kids, including our core beliefs about their psychological development. Take thetoddler. I assumed that phase was something experts develop

86、ed after years of researchinto childrens behaviour: wrong. Turns out, according to Daniel Cook, a historian ofchildhood consumerism, it was popularised as a marketing trick by clothing manufacturersin the 1930s.Trade publications counselled department stores that, in order to increase sales, theysho

87、uld create a “third stepping, stone“ between infant wear and older kids clothes. It wasonly after “toddler“ became a common shoppers* term that it evolved into a broadlyaccepted developmental stage. Splitting kids, or adults, into ever-tinier categories hasproved a sure-fire way to boost profits. An

88、d one of the easiest ways to segment a marketis to magnify gender differences - or invent them where they did not previously exist.26.By saying it is.the rainbow(Line 3, Para.1),the author means pi nk.Ashould not be the sole representation of girlhoodBshould not be associated with girls* innocenceCc

89、annot explain girls lack of imaginationDcannot influence girls lives and interests27.According to Paragraph 2, which of the following is true of colours?AColours are encoded in girls* DNA.BBlue used to be regarded as the colour for girls.CPink used to be a neutral colour in symbolising genders.DWhit

90、e is prefered by babies.28 .The author suggests that our perception of childrens psychological development wasmuch influenced by.Athe marketing of products for childrenBthe observation of childrens natureCresearches into childrens behaviorDstudies of childhood consumption29 .We may learn from Paragr

91、aph 4 that department stores were advised to.Afocus on infant wear and older kids* clothesBattach equal importance to different gendersCclassify consumers into smaller groupsDcreate some common shoppers terms30.l t can be concluded that girls* attraction to pink seems to be.A clearly explained by th

92、eir inborn tendencyBfully understood by clothing manufacturersC mainly imposed by profit-driven businessmenDwell interpreted by psychological expertsText3In 2010, a federal judge shook Americas biotech industry to its core. Companies had wonpatents for isolated DNA for decades-by 2005 some 20% of hu

93、man genes were pa rented.But in March 2010 a judge ruled that genes were unpatentable. Executives were violentlyagitated. The Biotechnology Industry Organisation (BIO), a trade group, assuredmembers that this was just a preliminary step in a longer battle.On July 29th they were relieved, at least te

94、mporarily. A federal appeals court overturnedthe prior decision, ruling that Myriad Genetics could indeed hold patents to two genes thathelp forecast a womans risk of breast cancer. The chief executive of Myriad, a companyin Utah, said the ruling was a blessing to firms and patients alike.But as com

95、panies continue their attempts at personalised medicine, the courts will remainrather busy. The Myriad case itself is probably not over Critics make three mainarguments against gene patents: a gene is a product of nature, so it may not be patented;gene patents suppress innovation rather than reward

96、it; and patents monopolies restrictaccess to genetic tests such as Myriads. A growing number seem to agree. Last year afederal task-force urged reform for patents related to genetic tests. In October theDepartment of Justice filed a brief in the Myriad case, arguing that an isolated DNAmolecule “is

97、no less a product of nature. than are cotton fibres that have beenseparated from cotton seeds. ”Despite the appeals courts decision, big questions remain unanswered. For example, it isunclear whether the sequencing of a whole genome violates the patents of individualgenes within it. The case may yet

98、 reach the Supreme Court.As the industry advances, however, other suits may have an even greater impact.Companies are unlikely to file many more patents for human DNA molecules-most arealready patented or in the public domain .firms are now studying how genes interact,looking for correlations that m

99、ight be used to determine the causes of disease or predict adrugs efficacy, companies are eager to win patents for connecting the dots/* explainsHans Sauer, a lawyer for the BIO.Their success may be determined by a suit related to this issue, brought by the MayoClinic, which the Supreme Court will h

100、ear in its next term. The BIO recently held aconvention which included sessions to coach lawyers on the shifting landscape forpatents .Each meeting was packed.31. It can be learned from Paragraph 1 that the biotech companies would likeA their executives to be activeB judges to rule out gene patentin

101、gC genes to be patentableD the BIO to issue a warning32. Those who are against gene patents believe that .A genetic tests are not reliableB only man-made products are patentableC patents on genes depend much on innovationD courts should restrict access to gene tic tests33. According to Hans Sauer, c

102、ompanies are eager to win patents for .A establishing disease correlationsB discovering gene interactionsC drawing pictures of genesD identifying human DNA34. By saying “each meeting was packed,(Line4,Para.6) the author means thatA the supreme court was authoritativeB the BIO was a powerful organiza

103、tionC gene patenting was a great concernD lawyers were keen to attend conventions35. Generally speaking, the authors attitude toward gene patenting isA criticalB supportiveC scornfulD objectiveText 4The great recession may be over, but this era of high joblessness is probably beginning.Before it end

104、s, it will likely change the life course and character of a generation of youngadults. And ultimately, it is likely to reshape our politics, our culture, and the character ofour society for years.No one tries harder than the jobless to find silver linings in this national economic disaster.Many said

105、 that unemployment, while extremely painful, had improved them in some ways;they had become less materialistic and more financially prudent; they were more aware ofthe struggles of others. In limited respects, perhaps the recession will leave society betteroff. At the very last, it has awoken us fro

106、m our national fever dream of easy riches andbigger houses, and put a necessary end to an ear of reckless personal spending.But for the most part, these benefits seem thin, uncertain, and far off. In The MoralConsequences of Economic Growth, the economic historian Benjamin Friedman arguesthat both i

107、nside and outside the U.S., lengthy periods of economic stagnation or declinehave almost always left society more mean-spirited and less inclusive, and have usuallystopped or reversed the advance of rights and freedoms. Anti-immigrant sentimenttypically increases, as does conflict between races and

108、classes.In come inequality usually falls during a recession, but it has not shrunk in this one. Indeed,this period of economic weakness may reinforce class divides, and decreaseopportunities to cross them-especially for young people. The research of Till Von Wachter,the economist at Columbia Univers

109、ity suggests that not all people graduating into arecession see their life chances dimmed; those with degrees from elite universities catchup fairly quickly to where they otherwise would have been if they had graduated in bettertimes; it is the masses beneath them that are left behind.In the Interne

110、t age, it is particularly easy to see the resentment that has always beenhidden within American society. More difficult, in the moment, is discerning precisely howthese lean times are affecting societ/s character. In many respects, the U.S. was moresocially tolerant entering this recession than at a

111、ny time in its history, and a variety ofnational polls on social conflict since then have shown mixed results. We will have to waitand see exactly how these hard times will reshape our social fabric. But they certainly it,and all the more so the longer they extend.36. By saying “to find silver linin

112、gs5, (Line 1, Para. 2) the author suggests that the joblesstry to .A seek subsidies from the governmentB explore reasons for the unemploymentC make profits from the troubled economyD look on the bright side of the recession37. according to Paragraph 2, the recession has made peopleA realize the nati

113、onal dreamB struggle against each otherC challenge their prudenceD reconsider their lifestyle38. Benjamin Friedman believes that economic recession mayA impose a heavier burden on immigrantsB bring out more evils of human natureC promote the advance of rights and freedomsD ease conflicts between rac

114、es and classes39.The research of Till Von Wachter suggests that in the recession graduates from eliteuniversities tend toA lag behind the others due to decreased opportunitiesB catch up quickly with experienced employeesC see their life chances as dimmed as the othersD recover more quickly than the

115、others40. The author thinks that the influence of hard times on society isA certainB positiveC trivialD destructivePart BDirections:Read the following text and answer the questions by finding information from the rightcolumn that corresponds to each of the marked details given in the left column. Th

116、ere aretwo extra choices in the right column. Mark your answer on ANSWER SHEET 1. (10points)“Universal history, the history of what man has accomplished in this world, is at bottomthe History of the Great Men who have worked here, wrote the Victorian sage ThomasCarlyle. Well, not any more it is not.

117、Suddenly, Britain looks to have fallen out with its favourite historical form. This could beno more than a passing literary craze, but it also points to a broader truth about how wenow approach the past: less concerned with learning from forefathers and more interestedin feeling their pain. Today, w

118、e want empathy, not inspiration.From the earliest days of the Renaissance, the writing of history meant recounting theexemplary lives of great men. In 1337, Petrarch began work on his rambling writing DeViris lllustribus - On Famous Men, highlighting the virtus (or virtue) of classical heroes.Petrar

119、ch celebrated their greatness in conquering fortune and rising to the top. This wasthe biographical tradition which Niccolo Machiavelli turned on its head. In The Prince, thechampioned cunning,ruthlessness, and boldness, rather than virtue, mercy and justice, as the skills ofsuccessfulleaders.Over t

120、ime, the attributes of greatness shifted. The Romantics commemorated theleading painters and authors of their day, stressing the uniqueness of the artists personalexperience rather than public glory. By contrast, the Victorian author Samual Smiles wroteSelf-Help as a catalogue of the worthy lives of

121、 engineers , industrialists and explores .*The valuable examples which they furnish of the power of self-help, if patient purpose,resolute working and steadfast integrity, issuing in the formulation of truly noble and manycharacter, exhibit,wrote Smiles.what it is in the power of each to accomplish

122、forhimselfHis biographies of James Walt,Richard Arkwright and Josiah Wedgwood were held up as beacons to guide the workingman through his difficult life.This was all a bit bourgeois for Thomas Carlyle, who focused his biographies on the trulyheroic lives of Martin Luther, Oliver Cromwell and Napoleo

123、n Bonaparte. These epochalfigures represented lives hard to imitate, but to be acknowledged as possessing higherauthority than mere mortals.Communist Manifesto. For them, history did nothing, it possessed no immense wealthnor waged battles:ult is man, real, living man who does all that. And history

124、should be thestory of the masses and their record of struggle. As such, it needed to appreciate theeconomic realities, the social contexts and power relations in which each epoch stood.For:uMen make their own history, but they do not make it just as they please; they do notmake it under circumstance

125、s chosen by themselves, but under circumstances directlyfound, given and transmitted from the past.”This was the tradition which revolutionized our appreciation of the past. In place ofThomas Carlyle, Britain nurtured Christopher Hill, EP Thompson and Eric Hobsbawm.History from below stood alongside

126、 biographies of great men. Whole new realms ofunderstanding - from gender torace to cultural studies - were opened up as scholarsunpicked the multiplicity of lost societies. And it transformed public history too: downstairsbecame just as fascinating as upstairs.A emphasized the virtue of classical h

127、eroes.41. PetrarchB highlighted the public glory of the leading artists.42. Niccolo MachiavellliC focused on epochal figures whose lives were hard toimitate.43. Samuel SmilesD opened up new realms of understanding the great men inhistory.44. Thomas CarlyleE held that history should be the story of t

128、he masses andtheir record of struggle.45. Marx and EngelsF dismissed virtue as unnecessary for successful leaders.G depicted the worthy lives of engineer industrialists andexplorers.Section III Translation46. Directions:Translate the following text from English into Chinese. Write your translation o

129、n ANSWERSHEET 2. (15 points)When people in developing countries worry about migration, they are usually concernedat the prospect of their best and brightest departure to Silicon Valley or to hospitals anduniversities in the developed world. These are the kind of workers that countries likeBritian, C

130、anada and Australia try to attract by using immigration rules that privilegecollege graduates.Lots of studies have found that well-educated people from developing countries areparticularly likely to emigrate. A big survey of Indian households in 2004 found that nearly40% of emigrants had more than a

131、 high-school education, compared with around 3.3%ofall Indians over the age of 25 . This brain drain has long bothered policymakers in poorcountries. They fear that it hurts their economies, depriving them of much-needed skilledworkers who could have taught at their universities, worked in their hos

132、pitals and come upwith clever new products for their factories to make.Section IV WritingPart A47. Directions:Suppose you have found something wrong with the electronic dictionary thatyou bought from an online store the other day .Write an email to the customer servicecenter to1) make a complaint an

133、d2) demand a prompt solutionYou should write about 100 words on ANSERE SHEET 2.Do not sign your own name at the end of the letter, Use “zhang wei instead.Do not write the address. (10 points)Part B48. Directions:Write an essay based on the following table. In your writing, you should1) describe the

134、table ,and2) give your commentsYou should write at least 150 words(15points)Write your easy on ANSERE SHEET 2.某公司员工工作满意度调查满意度年龄组满意不清楚不满意4。 岁16.7%50.0%33. 3%415。 岁0.0%36.0%64.0%5。 岁40.0%50.0%10.0%客观题答案1-5 BBAAC 6-10 DCACB 11-15 DBCDD16-20 ACCBD 21-25 ACABD 26-30 ABACC31-35 CBBCD 36-40 DDBDA 41-45 AFG

135、CE46 . 翻译答案:当来自发展中国家的人们担心移民的问题时,他们通常关心的是自己是否有希望离开家乡,而成功地进入硅谷,或进入发达国家的医院和大学工作。这些人才是诸如英国、加拿大和澳大利亚等国期望吸引到的类型,这些国家通常制定吸引大学毕业生的移民政策。很多研究表明,来自发展中国家的受过良好教育之人特别容易移民。2004年,一份对印度家庭的大型调查中,发现将近40%的移民者受教育程度在高中以匕 而相比之下,整个印度超过25岁的居民中,只有大约3.3%的人受过高中以上的教育程度。这样的“ 人才流失”现象长期困扰着不发达国家的政策制定者。 他们担心这样会损害他们的经济, 夺走本国急需的人才, 而这

136、些人原本可以在本国的大学教书、 在医院工作, 以及为本国工厂开发新型产品。小作文范文:Dear Sir or Madame,As one of the regular customers of your online store, I am writing this letter to express mycomplaint against the flaws in your productan electronic dictionary I bought in your shopthe other day.The dictionary is supposed to be a favorabl

137、e tool for my study. Unfortunately, I found thatthere are several problems. To begin with, when I opened it, I detected that theappearance of it had been scratched. Secondly, I did not find the battery promised in theadvertisement posted on the homepage of your shop, which makes me feel that you hav

138、enot kept your promise. What is worse, some of the keys on the keyboard do not work.I strongly request that a satisfactory explanation be given and effective measures shouldbe taken to improve your service and the quality of your products. You can either send anew one to me or refund me my money in

139、full.I am looking forward to your reply at your earliest convenience.Sincerely yours,Zhang Wei( 三)2011年 MBA综合真题及答案一、问题求解:第 115小题,每小题3 分,共 4 5 分。下列每题给出的A、B、C、D、E五个选项中,只有一项是符合要求的。请在答题卡上将所选项的字母涂黑。1.已知船在静水中的速度为28km /h,河水的流速为2km /h,则此船在相距78km的两地间往返一次所需时间是(A) 5.9h (B) 5.6h (C) 5.4h (D) 4.4h (E) 4h2. 若实数 a

140、, b, c 满足 |a-3| + J 3b+ 5 + (5c-4)2 = 0,贝 ijabc =(A)-4(B )-14 4(C )-y (D )-(E)33. 某年级60名学生中,有 30人参加合唱团、45人参加运动队,其中参加合唱团而未参加运动队的有8 人,则参加运动队而未参加合唱团的有(A) 15 人 (B )22 人 (0 23 人 (D )30 人 (E )37 人4. 现有一个半径为R的球体,拟用刨床将其加工成正方体,则能加工的最大正方体的体积是(A )-R3 (B ) R33 9(C)(R3 (D)1R3(E) R395.2007年,某市的全年研究与试验发展(R & D )经费

141、支出300亿元,比 2006年增长20%,该市的GDP为 10000亿元,比 2006年增长10%。 2006年,该市的R&D经费支出占当年GDP的(A) 1.75% (B) 2% (C) 2.5% (D) 2.75% (E) 3%6. 现从5名管理专业,4名经济专业和1 名财会专业的学生中随机派出一个3人小组,则该小组中3个专业各有1 名学生的概率为(A )- (B )-2 3( O -(D )-4 57. 一所四年制大学每年的毕业生七月份离校,新生九月份入学。该校2001年招生2000名,之后每年比上一年多招200名,则该校2007年九月底的在校学生有(A)14000名 (B)11600名

142、(09000名 (D )6200名(E )3200名8 . 将 2个红球与1个白球随机地放入甲、乙、丙三个盒子中,则乙盒中至少有1 个红球的概率为9 . 如图L 四边形ABCD是边长为1 的正方形,弧AOB, BOC, COD, DOA均为半圆,则阴影部分的面积为 .(A )-210 .3个3口之家一起观看演出,他们购买了同一排的9张连座票,则每一家的人都坐在一起的不同坐法有(A)(3!)2 种(03(3!) 3 种(E)9!种(B)(3!)3 种(D)(3!)4 种11 . 设P是圆x2 + y2 =2上的一点,该圆在点P的切线平行于直线x + y + 2 = 0,则点P的坐标为(C)(0,

143、72(E)(l,l)(D)(V2,0)12 . 设 a, b,c是小于12的三个不同的质数(素 数 ),fi|a - b| + |b - c| + |c - a| = 8 , 则a + b + c =(A)10 (B)12 (C)14 (D)15 (E)1913 . 在年底的献爱心活动中,某单位共有100人参加捐款,经统计,捐款总额是19000元,个人捐款数额有100元、500元和2000元三种,该单位捐款500元的人数为(A)13 (B)18 (C)25 (D)30 (E)2814 . 某施工队承担了开凿一条长为2400m隧道的工程, 在掘进了 400m后,山于改进了施工工艺,每天比原计划多

144、掘进2 m ,最后提前50天完成了施工任务,原计划施工工期是(A)200 天 (B)240 天 (C)250 天 (D)300 天 (E)350 天15 .已知x 2 + y2 = 9 ,x y = 4,贝 l j -=x , + y5 + x + y( 呜(B )l : ( 呜(E )14二 条件充分性判断: 第 16-25小题, 每小题3 分, 共 30分。 要求判断每题给出的条件(1)和 条 件 ( 2 ) 能否充分支持题干所陈述的结论,A、B、C、D、E 五个选项为判断结果,请选择项符合试题要求的判断,在答题卡上将所选项的字母涂黑。( A ) 条 件(1 ) 充分,但 条 件 ( 2

145、) 不充分。(B ) 条 件(2 ) 充分,但 条 件(1 ) 不充分。(C ) 条 件(1 ) 和(2 ) 单独都不充分,但 条 件(1 ) 和 条 件 ( 2 ) 联合起来充分。( D ) 条 件(1 ) 充分,条 件(2 ) 也充分。( E ) 条 件(1 ) 和(2 ) 单独都不充分,条 件(1 ) 和(2 ) 联合起来也不充分。16 . 实数a,b,C成等差数列。(1)ea,eb,e,成等比数列(2) Ina, Inb, Inc 成等差17 .在一次英语考试中,某班的及格率为80%.( 1 ) 男生及格率为7 0 % ,女生及格率为90%.(2 ) 男生的平均分与女生的平均分相等.1

146、8 . 如图2 , 等腰梯形的上底与腰均为X,下底为x+10.则 x=13.( 1 ) 该梯形的上底与下底之比为13: 23.( 2 ) 该梯形的面积为216.图19 .现有3名男生和2名女生参加面试则面试的排序法有24种 .(1 )第一位面试的是女生(2 )第二位面试的是指定的某位男生.20 .已知三角形A B C的三条边分别为a, b, c.则三角形ABC是等腰直角三角形(1)(a-b)(c2- a2-/72)=0(2) c= 4 2 b21 .直线ax+ by+ 3=0被圆(x - 2)2 + ( y - l)2 =4截得的线段的长度为2 V3 a=0, b=- 1 a=- 1, b=0

147、22.已知实数 a, b, c, d满足屋 + b = l,c2 + d2 = 1 . 则+ 卜 1(1 )直线ax+ by=1与cx+ dy=1仅有一个交点(2) a。c, b片 d23 .某年级共有8 个班友,在一次年级考试中,共 有 2 1 名学生不及格,每班不及格的学生最多有3 名,则 (一 )班 至 少 有 1 名学生不及格(1 )( - ) 班的不及格人数多于(三)班(2 )(四)班不及格的学生有2 名24 .现有一批文字材料需要打印, 两台新型打印机单独完成此任务分别需要4 小时与5 小时,两台旧型打印机单独完成此任务分别需要9 小时与11小时,则能在2.5小时内完成任务(1 )

148、安排两台新型打印机同时打印(2 )安排一台新型打印机与两台旧型打印机同时打印25 .已知 a“ 为等差数列,则该数列的公差为零(1 )对任何正整数n, Mal+a2+.+a a226 .巴斯德认为,空气中的微生物浓度与环境状况、气流运动和海拔高度有关。他在山上的不同高度分别打开装着煮过的培养液的瓶子, 发现海拔越来越高, 培养液被微生物污染的可能性越小。在山顶上,2 0 个装了培养液的瓶子,只 有 1 个长出了微生物。普歇另用干草浸液做材料重复了巴斯德的实验,却得出不同的结果:及时在海拔很高的地方,所有装了培养液的瓶子很快长出了微生物。以下哪项如果为真,最能解释普歇和巴斯德实验所得到的不同结果

149、?( A ) 只要有氧气的刺激,微生物就会从培养液中自发地生长出来。(B)培养液在加热消毒、密封、冷却的过程中会被外界细菌污染。(C)普歇和巴斯德的实验设计都不够严密。(D)干草浸液中含有一种耐高温的枯草杆菌, 培养液旦冷却, 枯草杆菌的抱子就会复活,迅速繁殖。(E)普歇和巴斯德都认为,虽然他们用的实验材料不同,但是经过煮沸,细菌都能被有效地杀灭。27 . 张教授的所有初中同学都不是博士,通过张教授而认识其哲学研究所同事的都是博士;张教授的一个初中同学通过张教授认识了王研究员。以下哪项能作为结论从上述中退出?(A )王研究员是张教授的哲学研究所同事(B)王研究员不是张教授的哲学研究所同事(C)

150、我那个研究员是博士(D)王研究员不是张教授的初中同学28. 一般将缅甸所产的经过风化或经河水搬运至河谷、河床中的西翠大砾石,称为“ 老坑玉” 。老坑玉的特点是“ 水头好” 、质 坚 、透明度高,其上品透明如玻璃,故称“ 玻璃种” 或“ 冰种” 。同为老坑玉,其质量相对也有高低之分,有 的 透 明 度 高 些 ,有的透明度稍差一些,所以价值也有差别。在其他条件都相同的情况下,透明度高的老坑玉比透明度较其低的单位价值高,但是开采的实践告诉人们,没有单位价值最高的老坑玉。以上论述如果为真,可以得出以下哪项结论?(A)没有透明度最高的老坑玉(B)透明度最高的老坑玉未必“ 水 头好”(C) “ 老坑玉”

151、 中也有质量好的翡翠(D)老坑玉的单位价值还决定于其加工的质量(E)随着年带的增加,老坑玉的单位价值会越来越高29 . 某教育专家认为:“ 男孩危机” 是指男孩调皮捣蛋、胆小怕事、学习成绩不如女孩好等现象 。近 些 年 ,这种现象已经成为儿童教育专家关注的一个重要问题。这位专家在列出一系列统计数据后,提出了“ 今日男孩为什么从小学、中学到大学全面落后于同年龄阶段的女孩” 的疑问,这无疑加剧了无数男孩家长的焦虑。该专家通过分析指出,恰恰是家庭和学校不恰当的教育方法导致了“ 男孩危机” 现 象 。以下哪项如果为真,最能对该专家的观点提出质疑?(A)家庭对独生子女的过度呵护, 在很大程度上限制了男孩

152、发散思维的拓展和冒险性格的养 成 。(B)现在的男孩比以前的男孩在女孩面前更喜欢表现出“ 绅士” 的一面。(C)男孩在发展潜能方面要优于女孩,大学毕业后后他们更容易在事业上有所成就。(D)在 家 庭 、学校教育中,女性充当了主要角色。(E)现代社会游戏泛滥,男孩天性比女孩更喜欢游戏,这耗去了他们大量的精力。30 .抚仙湖虫是泥盆纪澄江动物群中特有的一种,属于直节肢动物中比较原始的类型,成虫体 长10厘 米 ,有31个体节,外骨骼分为头、胸 、腹三部分,它的背、腹分节不一致。泥盆纪直虾是现代昆虫的祖先,抚仙湖虫化石与直虾类化石类似,这间接表明了抚仙湖虫是昆虫的远祖。研究者还发现,抚仙湖虫的消化道

153、充满泥沙,这表明它是食泥动物。以下除哪项外,均能支持上述论证?( A )昆虫的远祖也有不食泥的生物。( B )泥盆纪直虾的外骨骼分为头、胸 、腹三部分。( C )凡是与泥盆纪直虾类似的生物都是昆虫的远祖。( D )昆虫是山真节肢动物中比较原始的生物进化而来的。( E )抚仙湖虫消化道中的泥沙不是在化石形成过程中有外界渗透进去的。31. 2010年 某省物价总水平仅上涨2.4% ,涨势比较温和,涨 幅 甚 至 比2009年回落了 0.6个百分点。可 是 ,普通民众觉得物价涨幅较高,些统计数据也表明,民众的感觉有据可依 。2010年某月的统计报告显示,该月禽蛋类食品价格涨幅达12.3% ,某些反季

154、节蔬菜涨 幅 甚 至 超 过20% o以下哪项如果为真,最能解释上述看似矛盾的现象?( A )人们对数据的认识存在偏差,不同来源的统计数据会产生不同的结果。( B )影响居民消费品价格总水平变动的各种因素互相交织。( C ) 虽然部分日常消费品涨幅很小,但居民感觉很明显。( D ) 在物价指数体系中占相当权重的工业消费品价格持续走低。(E ) 不同的家庭,其收入水平、消费偏好,消费结构都有很大差异。32 . 随着互联网的发展, 人们的购物方式有了新的选择。 很多年轻人喜欢在网络上选择自己满意的商品,通过快递送上门,购物足不出户,非常便捷。刘教授据此认为,那些实体商场的竞争力会受到互联网的冲击,

155、在不远的将来,会有更多的网络商店取代实体商店。以下哪项如果为真,最能削弱刘教授的观点?( A ) 网络购物虽然有某些便利,但容易导致个人信息被不法分子利用。( B ) 有些高档品牌的专卖店,只愿意采取街面实体商店的销售方式。(C ) 网络商店与快递公司在货物丢失或损坏的赔偿方面经常互相推诿。( D ) 购买黄金珠宝等贵重物品,往往需要现场挑选,且不适宜网络支付。(E ) 通常情况下,网络商店只有在其实体商店的支持下33 . 受多元文化和价值观的冲击,中国居民的离婚率明显上升。最近一项调查表明,中国的平均婚姻持续时间为8 年。张先生为此感慨,现在像钻石婚、金婚、白头偕老这样的美丽故事已经很难得,

156、人们纯朴的爱情婚姻观- 去不复返了。以下哪项如果为真,最可能表明张先生的理解不确切?(A)现在有不少闪婚族,他们经常在很短的时间里结婚又离婚。(B)婚姻存续时间长并不意味着婚姻的质量高。(C)过去的婚姻主要由父母包办,现在主要是自由恋爱。(D)尽管婚姻存续时间短,但年轻人谈恋爱的时间比以前增加很多。(E)婚姻是爱情的坟墓,美丽感人的故事更多体现在恋爱中。34 . 某集团公司有四个部门,分别生产冰箱、彩电、电脑和手机。根据前三个季度的数据统计,四个部门经理对2010年全年的赢利情况作了如下预测:冰箱部门经理彩电部门经理电脑部门经理手机部门经理今年手机部门会赢利。如果冰箱部门今年没赢利,那么彩电部

157、门就不会赢利。如果手机部门今年没赢利,那么电脑部门也没赢利。今年冰箱和彩电部门都会赢利。全年数据统计完成后,发现上述四个预测只有一个符合事实。(A)彩电部门赢利,冰箱部门没赢利。(B)冰箱部门赢利,电脑部门没赢利。(C)电脑部门赢利,彩电部门没赢利。(D)冰箱部门和彩电部门都没赢利。(E)冰箱部门和电脑部门都赢利。35 . 随着数字技术的发展,音频、视频的播放形式出现了革命性转变。人们很快接受了一些新形式,比 如 MP3、CD、DVD等。但是对于电子图书的接受并没有达到专家所预期的程度, 现在仍有很大一部分读者喜欢捧着纸质出版物。 纸质书籍在出版业中依然占据重要地位。因此有人说,书籍可能是数字

158、技术需要攻破的最后一个堡垒。以下哪项最不能对上述现象提供解释?(A)人们固执地迷恋着阅读纸质书籍时的舒适体验,喜欢纸张的质感。(B)在显示器上阅读,无论是笨重的阴极射线管显示器还是轻薄的液晶显示器,都会让人无端地心浮气躁。(C)现在仍有一些怀旧爱好者喜欢收集经典图书。(D)电子书显示设备技术不够完善,图像显示速度较慢。(E)电子书和纸质书籍的柔软沉静相比,显得面目可憎。36. 在一次围棋比赛中,参赛选手陈华不时地挤捏指关节,发出声响干扰了对手的思考。在比赛封盘间歇时, 裁判警告陈华, 如果再次在比赛中挤捏指关节并发出声响, 将判其违规。对此,陈华反驳说,他挤捏指关节是习惯性动作,并不是故意的,

159、因此,不应被判违规。以下哪项如果成立,最能支持陈华对裁判的反驳?(A)在此次比赛中,对手不时打开、合拢折扇,发出的声响干扰了陈华的思考。(B)在围棋比赛中,只有选手的故意行为,才能成为判罚的根据。(C)在此次比赛中,对手本人并没有对陈华的干扰提出抗议。(D)陈华一向恃才傲物,该裁判对其早有不满。(E)如果陈华为人诚实、从不说谎,那么他就不应该被判违规。37.3D立体技术代表了当前电影技术的尖端水准,由于使电影实现了高度可信的空间感,它可能成为未来电影的主流。3D立体电影中的银幕角色虽然由计算机生成,但是那些包括动作和表情的电脑角色的“ 表演” ,都以真实演员的“ 表演” 为基础,就想数码时代的

160、化妆技术一样。这也引起了某些演员的担心:随着计算机技术的发展,未来计算机生成的图像和动画会代替真人表演。以下哪项如果为真,最能减弱卜一述演员的担心?(A)所有电影的导演只能和真人交流,而不是和电脑交流。(B)任何电影的拍摄都取决于制片人的选择,演员可以跟上时代的发展。(C)3D立体电影目前的高票房只是人们一时图新鲜的结果,未来尚不可知。(D)掌 握3D立体技术的动画专业人员不喜欢去电影院看3D电影。(E)电影故事只能用演员的心灵、情感来表现,其表现形式与导演的喜好无关。38 . 公达律师事务所以为刑事案件的被告进行有效辩护而著称,成功率达90%以上。老余是一位以专门为离婚案件的当事人成功辩护而

161、著称的律师。因此, 老余不可能是公达律师事务所的成员。以下哪项最为确切地指出了上述论证的漏洞?(A)公达律师事务所具有的特征,其成员不一定具有。(B)没有确切指出老余为离婚案件的当事人辩护的成功率。(C)没有确切指出老余为刑事案件的当事人辩护的成功率。(D)没有提供公达律师事务所统计数据的来源。(E)老余具有的特征,其所在工作单位不一定具有。39 .科学研究中使用的形式语言和I I常生活中使用的自然语言有很大的不同, 形式语言看起来像天书,远离大众,只有一些专业人士才能理解和运用。但其实这是一种误解,自然语言和形式语言的关系就像肉眼与显微镜的关系,肉眼的视域广阔, 可以从整体上把握事物的信息;

162、 显微镜可以帮助人们看到事物的细节和精微之处, 尽管用它看到的范围小, 所以,形式语言和自然语言都是人们交流和理解信息的重要工具, 把它们结合起来使用, 具有强大的力量。以下哪项如果为真,最能支持上述结论?(A )通过显微镜看到的内容肯能成为新的“ 风景” , 说明形式语言可以丰富自然语言的表达,我们应重视形式语言。( B ) 正如显微镜下显示的信息最终还是要通过肉眼观察一样,形式语言表述的内容最终也要通过自然语言来实现,说明自然语言更基础。( C ) 科学理论如果仅用形式语言表达,很难被普通民众理解;同样,如果仅用自然语言表达,有可能变得冗长且很难表达准确。( D ) 科学的发展很大程度上改

163、善了普通民众的日常生活,但人们并没有意识到科学表达的基础一一形式语言的重要性。( E ) 采用哪种语言其实不重要,关键在于是否表达了真正想表达的思想内容。40. 一艘远洋帆船载着5 位中国人和几位外国人由中国开往欧洲。途中,除 5 位中国人外,全患上了败血症。同乘一艘船,同样是风餐露宿,漂洋过海,为什么中国人和外国人如此不同呢?原 来 这 5 位中国人都有喝茶的习惯,而外国人却没有。于是得出结论:喝茶时这 5 位中国人未得败血症的原因。以下哪项和题干中得出结论的方法最为相似?( A ) 警察锁定了犯罪嫌疑人,但是从目前掌握的事实看,都不足以证明他犯罪。专案组由此得出结论,必有一种未知的因素潜藏

164、在犯罪嫌疑人身后。( B ) 在两块土壤情况基本相同的麦地上,对其中一块施氮肥和钾肥,另一块只施钾肥。结果施氮肥和钾肥的那块麦地的产量远高于另一块。可见,施氮肥是麦地产量较高的原因。(C ) 孙悟空:“ 如果打白骨精,师父会念紧箍咒;如果不打,师父就会被妖精吃掉。” 孙悟空无奈得出结论:“ 我还是回花果山算了。”( D ) 天文学家观测到天王星的运行轨道有特征a、b、c , 已知特征a、b 分别是由两颗行星甲、乙的吸引造成的,于是猜想还有一颗未知行星造成天王星的轨道特征c。(E) 一定压力下的一定量气体,温度升高,体积增大;温度降低,体积缩小;气体体积与温度之间存在一定得相关性,说明气体温度的

165、改变是它体积改变的原因。41 . 所有重点大学的学生都是聪明的学生,有些聪明的学生喜欢逃学,小杨不喜欢逃学;所以,小杨不是重点大学的学生。以下哪项除外,均与上述推理的形式类似?( A ) 所有经济学家都懂经济学,有些懂经济学的爱投资企业,你不爱投资企业,所以,你不是经济学家。(B )所有鹅都爱吃青菜,有些吃青菜的也吃鱼,兔子不吃鱼;所以,兔子不是鹅。(C)所有的人都是爱美的, 有些爱美的还研究科学, 亚里士多德不是普通人; 所以, 亚里士多德不研究科学。(D)所有被高校录取的学生都是超过录取分数线的,有些超过录取分数线的是大龄考生,校长不是大龄考生;所以小张没有被高校录取。(E)所有想当外交官

166、的都需要学外语,有些学外语的重视人际交往,小王不重视人际交往;所以小王不想当外交官。42 . 按照联合国开发计划署2007年的统计,挪威是世界上居民生活质量最高的国家,欧美和 II本等发达国家也名列前茅。如果统计1990年以来生活质量改善最快的国家,发达国家则落后了,至少在联合国开发计划署统计的116个国家中,17年来,非洲东南部国家莫桑比克的生活质量提高最快,2007年其生活质量指数比1990年提高了 5 0 % ,很多非洲国家取得了和莫桑比克类似的成就。 作为世界上最受瞩目的发展中国家, 中国的生活质量指数在过去17年中冶提高了 27%。以下哪项可以从联合国开发计划署的统计中得出?(A)2

167、007年,发展中国家的生活质量指数都低于西方国家(B)2007年,莫桑比克的生活质量指数不高于中国(C)2006年,日本的生活质量指数不高于中国(D)2006年,莫桑比克的生活质量的改善快于非洲其他各国(E)2007年,挪威的生活质量指数高于非洲各届。43 . 某次认知能力测试,刘强得了 118分,蒋明的得分比王丽高,张华和刘强的得分之和大于蒋明和王丽的得分之和,刘强的得分比周梅高;此次测试120分以上为优秀,无人之中有两人没有达到优秀、根据以上信息,以卜哪项是上述五人在此次测试中得分由高到低的排列?(A)张华、王丽、周梅、蒋明、刘强 (B)张华、蒋明、王丽、刘强、周梅(C)张华、蒋明、刘强、

168、王丽、周梅 (D)蒋明、张华、王丽、刘强、周梅(E)蒋明、王丽、张华、刘强、周梅44 . 近 I I , 某集团高层领导研究了发展方向问题。王总经理认为:既要发展纳米技术,也要发展生物医药技术;赵副总经理认为:只有发展智能技术,才能发展生物医药技术:李副总经理认为:如果发展纳米技术和生物医药技术,那么也要发展智能技术。最后经过董事会研究,只有其中一位的意见被采纳。根据以上陈述,以下哪项符合董事会的研究决定?(A ) 发展纳米技术和智能技术,但是不发展生物医药技术。( B ) 发展生物医药技术和纳米技术,但是不发展智能技术。(C ) 发展智能技术和生物医药技术,但是不发展纳米技术.( D ) 发

169、展智能技术,但是不发展纳米技术和生物医药技术。(E ) 发展生物医药技术,智能技术和纳米技术。45 . 国外某教授最近指出,长着一张娃娃脸的人意味着他将享有更长的寿命,因为人们的生活状况很容易反映在脸上,从 1990年春季开始,该教授领导的研究小组对1826对 70岁以上的双胞胎进行了体能和认知测试, 并拍了他们的面部照片。 在不知道他们确切年龄的情况下,三名研究助手先对不同年龄组的双胞胎进行年龄评估,结果发现,即使是双胞胎,被猜出的年龄也相差很大。然后, 研究小组用若干年时间对这些双胞胎的晚年生活进行了跟踪调查,直至他们去世。调查表明:双胞胎中,外表年龄差异越大,看起来老的那个就越可能先去世

170、。以下哪项如果为真,最能形成对该教授调查结论的反驳?( A ) 如果把调查对象扩大到40岁以上的双胞胎,结果可能会有所不同。( B ) 三名研究助手比较年轻,从事该项研究的时间不长( C ) 外表年龄是每个人生活坏境、生活状况和心态的集中体现,与生命老化关系不大。(D ) 生命老化的原因在于细胞分裂导致染色体末端不断损耗。( E ) 看起来越老的人,在心里上般较为成熟,对于生命有更深刻的理解。46 .由于含糖饮料的卡路里含量高,容易导致肥胖,因此无糖饮料开始流行,经过一段时期的调查,李教授认为:无糖饮料尽管卡路里含量低,但并不意味它不会导致体重增加,因为无糖饮料可能导致人们对于甜食的高度偏爱,

171、 这意味着可能食用更多的含糖类食物。而且无糖饮料几乎没什么营养,喝得过多就限制了其它健康饮品的摄入,比如茶和果汁等。以下哪项如果为真,最能支持李教授的观点?(A )茶是中国的传统饮料,长期饮用有益健康。(B )有些瘦子也爱喝无糖饮料。(C )有些胖子爱吃甜食。(D)不少胖子向医生报告他们常喝无糖饮料。(E )喝无糖饮料的人很少进行健身运动。4 7 .只有公司相关部门的所有员工都考试合格了,该部门的员工才能得到年终奖金,财务部有些员工考试合格了;综合部所有员工都得到了年总奖金;行政部的赵强考试合格了。如果以上陈述为真,则以下哪项可能为真?I . 财务部员工都考评合格了。 I I .赵强得到了年终

172、奖金。I II.综合部有些员工没有考评合格。 IV .财务部员工没有得到年终奖金。(A)仅 I、II (B)仅n、iii (C)仅 i、n、w(D )仅 I、II、III (E )仅H、in、IV48 .随着文化知识越来越重要,人们花在读书上的时间越来越多,文人学子中近视患者的比例也越来越高。即便在城里工人、乡镇农民中,也能看到不少人戴近视眼镜。然而,在中国古代很少发现患有近视的文人学子,更别说普通老百姓了。以下除哪项外,均可以解释上述现象?(A )古时候,只有家庭条件好或者有地位的人才读得起书:即便读书,用在读书上的时间也很少,那种头悬梁、锥刺骨的读书人更是凤毛麟角。(B )古时交通和工具不

173、发达,出行主要靠步行,骑马,足量的运动对于预防近视有一定的作用(C )古人生活节奏慢,不用担心交通安全,所以即使患了近视,其危害也非常小。(D )古代自然科学不发达,那时学生读的书很少,主要是四书五经,一 本 论语要读好几年。(E )古人书写用的是毛笔,眼睛和字的距离比较远,写的字也相对大些49-50题基于以下题干:某家长认为,有想象力才能进行创造性劳动,但想象力和知识是天敌。人在获得知识的过程中,想象力会消失,因为知识符合逻辑,而想象力无章可循。换句话说,知识的本质是科学,想象力的特征是荒诞。人的大脑一山不容二虎:学龄前,想象力独占鳌头,脑子被想象力占据; 上学后, 大多数人的想象力被知识驱

174、逐出境。 他们成为知识但丧失了想象力。终身只能重复前人发现的人。49 .以下哪项是该家长论述所依赖的假设?I ,科学是不可能荒诞的,荒诞的就不是科学。II .想象力和逻辑水火不相容。III .大脑被知识占据后很难重新恢复想象力。(A)仅 i (B)仅 ii (C)仅 i 和 n (D)仅 n 和ni (E)i 、n 和in50 . 以下哪项与该家长的上述观点矛盾?(A )如果希望孩子能够进行创造性劳动,就不要送他们上学。(B)如果获得了足够知识,就不能进行创造性劳动。(C)发现知识的人是有一定想象力的。(D)有些人没有想象力,但能进行创造性劳动。(E)想象力被知识驱逐出境是一个逐渐的过程。51

175、 . 某公司总裁曾经说过:“ 当前任总裁批评我时,我不喜欢那感觉,因此,我不会批评我的继任者。”以下哪项最可能是该总裁上述言论的假设?(A )当遇到该总裁的批评时,他的继任者和他的感觉不完全一致。(B)只有该总裁的继任者喜欢被批评的感觉,他才会批评继任者。(C)如果该总裁喜欢被批评,那么前任总裁的批评也不例外。(D)该总裁不喜欢批评他的继任者,但喜欢批评其他人。(E)该总裁不喜欢被前任总裁批评,但喜欢被其他人批评。52 . 在恐龙灭绝6500万年后的今天,地球正面临着又一次物种大规模灭绝的危机。截至上个世纪末,全球大约有20%的物种灭绝。现在,大熊猫、西伯利亚虎、北美玳瑁、巴西红木等许多珍稀物

176、种面临着灭绝的危险。有三位学者对此作了预测。学者一:如果大熊猫灭绝,则西伯利亚虎也将灭绝;学者二:如果北美玳瑁灭绝,则巴西红木不会灭绝:学者三:或者北美玳瑁灭绝,或者西伯利亚虎不会灭绝。如果三位学者的预测都为真,则以下哪项一定为假?(A )大熊猫和北美玳瑁都将灭绝。(B)巴西红木将灭绝,西伯利亚虎不会灭绝。(C)大熊猫和巴西红木都将灭绝。(D)大熊猫将灭绝,巴西红木不会灭绝。(E)巴西纤. 木将灭绝,大熊猫不会灭绝。53 . 一些城巾,山于作息时间比较统一,加上机动车太多,很容易造成交通早高峰和晚高峰,市民们在高峰时间上下班很不容易,为了缓解人们上下班时间的交通压力,某政府顾问提议采取不同时间

177、段上下班制度,即不同单位可以在不同的时间段上下班。以下哪项如果为真,最可能是该顾问的提议无法取得预期效果( A )有些上班时间段与员工的用餐时间冲突。会影响他们的生活规律,从而影响他们的工作积极性。(B)许多上班时间段与员工的正常作息时间不协调,他们需要较长一段时间来调整适应,这段时间的工作效率难以保证。(C)许多单位的大部分工作通常需要员工们在一起讨论,集体合作才能完成。( D)该城市机动车数量持续增加,即使不在早晚高峰期,交通拥堵也时有发生。( E)有些单位员工的住处与单位非常近,步行即可上下班。54 . 统计数字表明,近年来,民用航空飞机的安全性有很大提高。例如,某国2008年每飞行 1

178、00万次发生恶性事故的次数为0.2次, 而 1989年 为 1.4次。 从这些年的统计数字看,民用航空恶性事故发生率总体呈下降趋势,由此看出,乘飞机出行越来越安全。以下哪项不能加强上述结论?(A )近年来,飞机事故中“ 死里逃生” 的几率比以前提高了。(B)各大航空公司越来越注意对机组人员的安全培训。(C )民用航空的空中交通控制系统更加完善。(D )避免“ 机鸟互撞” 的技术与措施日臻完善。(E )虽然飞机坠毁很可怕,但从统计数字上讲,驾车仍然要危险得多。5 5 .有医学研究显示,行为痴呆症患者大脑组织中往往含有过量的铝。同时有化学研究表明, 一种硅化合物可以吸收铝。陈医生据此认为,可以用这

179、种硅化合物治疗行为痴呆症。以下哪项是陈医生最可能依赖的假设?(A )行为痴呆症患者大脑组织的含铝量通常过高,但具体数量不会变化。(B )该硅化合物在吸收铝的过程中不会产生副作用。(C )用来吸收铝的硅化合物的具体数量与行为痴呆症患者的年龄有关。(D )过量的铝是导致行为痴呆症的原因,患者脑组织中的铝不是痴呆症引起的结果。(E )行为痴呆症患者脑组织中的铝含量与病情的严重程度有关。四、写作( 本大题共2小题,共6 5分,其中第5 6题3 0分,第5 7题3 5分。请写在答题纸相应的位置上。)56 . 论证有效性分析: 分析下述论证中存在的缺陷和漏洞, 选择若干要点, 写一篇600字左右的文章,对

180、该论证的有效性进行分析和评述。如果你要从股市中赚钱, 就必须低价买进股票, 高价卖出股票, 这是人人都明白的基本道理。但是问题的关键在于如何判断股票价值的高低。只有正确的判断股价的高低, 上述的基本道理才有意义,否则就毫无实用价值。股价的高低是一个相对的概念,只有通过比较才能显现。一般来说,要正确判断一支股票的高低,唯的途径就是看它的历史表现。但是有人在判断当前某一股票的高低时,不注重股票的历史表现,而是注重股票今后的走势,这是一种危险的行为,因为股票的历史表现是一种客观事实,客观事实具有无可争辩的确定性;股票的今后走势只是一种主观预测,主观预测具有极大的不确定性。我们怎么可以只凭主观预测而不

181、顾客观事实呢?再说,股价的未来走势充满各种变数,它的涨和跌不是必然的,而是或然的。我们只能借助概率进行预测。假如宏观经济、市场态势和个人股表现均好,它的上涨概率就大;假如宏观经济、市场态势和个股表现均不好,它的上涨概率就小;假如宏观经济、市场态势和个股表现不相一致,它的上涨概率就需要酌情而定。山此可见,要从股市获取利益,第一是要掌握股价涨跌的概率, 第二还是要掌握股价涨跌的概率, 第三也还是要掌握股价涨跌的概率。掌握了股价涨跌的概率,你就能赚钱;否则,你就会赔钱。57 . 论说文:根据以下材料,写一篇700字左右的论说文, 自拟题目。众所周知,人才是立国、富国、强国之本。如何使人才尽快的脱颖而

182、出,是一个亟待解决的问题。人才的出现有多重途径,其中有“ 拔尖” ,有“ 冒尖” 。拔尖是通过提拔而成为尖子,冒尖是指通过奋斗、取得成就而得到社会公认。有人认为我国当今某些领域的管理人才,拔尖的多而冒尖的少。答案一 、数学部分15 BACBD1620 AEDBC二 、逻辑部分2630 DEBDA4145 CEBCA三 、写作部分610 EBDED 1115 EDADC2125 BEDDC3135 DEBBC 36一0 BAACB4650 DCCED 5155 BCDED50题 辩 证 有 效 性 分 析 【 参考答案】1 . “要正确判断某一股票的价格高低,唯一的途径就是看它的历史表现” ,显

183、然欠妥当,股票价格的高低由多种要素影响构成,论证者显然忽略其他影响要素。2 . “ 只注重股票今后的走势,这是一种危险的行为。” 不必然推出“ 唯一的途径就是看它的历史表现” ,因为,投资者完全可以二者兼顾。3 . 股票的历史表现是 种客观事实” ,“ 股票的今后走势只是一种主观预测” ,不必然推出“ 我们怎么可以只凭主观预测而不顾客观事实呢? ” 的结论。论述者“ 忽略发展” ,历史的表现未必能够代表该股票未来的走势。所谓的客观事实是过去的,并不能完全代表未来的“ 客观事实,4 . “ 我们只能借助概率进行预测” ,与论证者原有的论证自相矛盾。5 . “ 宏观经济、市场态势和个股表现” 是股

184、票价格的决定因素的部分因素,还需要考虑其他 因 素 ,再有他们之间的关系未必是简单的线性关系。6 . “ 掌握了股价涨跌的概率,你就能赚钱; 否则,你就会赔钱。该论证欠妥当,概率具有或然性不具有必然性,因此得不出必然赚饯或赔钱的结论。57 . 论说文:根据下述材料,写 一 篇700字左右的论说文。题目自拟。众所周知,人才是立国、富 国 、强国之本,如何使人才尽快的脱颖而出。是一个亟待解决的问题、 人才的出现有多种途径, 其中有“ 拔尖” ,有“ 冒尖” 。 “ 拔尖” 是指被提拔而成为尖子,冒尖是指通过奋斗、取得成就而得到社会的公认。有人认为当今某些领域的管理人才,拔尖的多而冒尖的少。可以写两

185、个角度:- :从个人角度写如何成为冒尖人才,强调个人奋斗和主观能动性;二 :从人与人角度写如何培养冒尖人才,强调创新型人才的培养方法。60题 论 说 文 【 参考范文】也谈“ 拔尖” 与“ 冒尖”有人认为我国当今某些领域的管理人才拔尖的多, 冒尖的少。我赞同这个观点。钱老认为:现在中国没有完全发展起来,一个重要原因没有自己独特的创新的东西,老是“ 冒” 不出杰出人才。钱老的这段话则是对当前人才现状的精辟分析。“ 拔尖” 和“ 冒尖” 二者似乎相差不大,强调的都是突出、出众的意思。但 是 ,深入体味二者的区别,不难发现:二者的不同之处不在于“ 尖” ,而在于“ 尖” 是如何出来的? 这才是问题的

186、关键。社会的发展更需要“ 冒尖” 管理人才。企业管理专家、管理学者、管理精英似乎我们并不缺少管理人才, 但是看看我国的管理人才竞争力排行榜中的位置,不由我们对一些现象深思回味。拔尖强调外力使其成为“ 尖” , 是矮子里面“ 拔” 将军的结果; 冒尖是指通过人才自身的努力脱颖而出,比较强调内力的作用。知识经济时代是一个追求效率的年代,也是一个知识更新、技术更新、管理方法更新最快的时代。若要跟上时代步伐,在各方面创造出高效率 ,就必须主动发挥人才的聪明才智,挖掘创新潜力,走创新之路。培养“ 冒尖” 管理人才不能仅强调在口头匕更要落实在行动中。培养“ 冒尖” 管理人才关键需要有创新的思想。海尔以“

187、日事日毕,日事日高” 来激励员工,也就是今天的事今天做好,而且今天要比昨天做的好一些。 这种做法不仅提高了员工的工作效率, 而且创造了一大批不断超越自我的优秀员工及管理者。这是思想创新在培养“ 冒尖” 管理人才的体现。邓小平开创的“ 改革开放” 无疑是社会制度方面的创新和重大突破,他让我们的国家跟上了世界发展的脚步,同时在各行各、 业培养出许多“ 冒尖” 人才,这是制度创新在培养人才的卓越典范。如何培养“ 冒尖” 管理人才,我们肩负着中华复兴的重任,每一个人都无法逃避。( 四 )2011年MBA英语真题及答案Section I Use of EnglishDirections:Read the

188、 following text. Choose the best word(s) for each numbered black and mark A, B,C or D on ANSWER SHEET 1. (10 points)The Internet affords anonymity to its users, a blessing to privacy and freedom ofspeech. But that very anonymity is also behind the explosion of cyber-crime that has 1across the Web.Ca

189、n privacy be preserved 2 bringing safety and security to a world that seemsincreasingly 3 ?Last month, Howard Schmidt, the nations cyber-czar, offered the federal governmenta 4 to make the Web a safer place-a voluntary trusted identity* system that would bethe high-tech 5 of a physical key, a finger

190、print and a photo ID card, all rolled 6 one.The system might use a smart identity card, or a digital credential 7 to a specificcomputer .and would authenticate users at a range of online services.The idea is to 8 a federation of private online identity systems. User could 9which system to join, and

191、only registered users whose identities have been authenticatedcould navigate those systems. The approach contrasts with one that would require anInternet drivers license 10 by the government.Google and Microsoft are among companies that already have theseusingle sign-onnsystems that make it possible

192、 for users to 11 just once but use many differentservices.12 .the approach would create a “walled garden n cyberspace, with safe“neighborhoods“ and bright “streetlights“ to establish a sense of a 13 community.Mr. Schmidt described it as a voluntary ecosystemn in which individuals andorganizations ca

193、n complete online transactions with 14 ,trusting the identities of eachother and the identities of the infrastructure 15 which the transaction runs.Still, the administrations plan has 16 privacy rights activists. Some applaud theapproach; others are concerned. It seems clear that such a scheme is an

194、 initiative pushtoward what would 17 be a compulsory Internet “drives license mentality.The plan has also been greeted with 18 by some computer security experts, whoworry that the “voluntary ecosystem envisioned by Mr. Schmidt would still leave much ofthe Internet 1 9 .They argue that all Internet u

195、sers should be 20 to register andidentify themselves, in the same way that drivers must be licensed to drive on publicroads.1.A.sweptB.skippedC.walkedD.ridden2.A.forB.withinC.while0.though3.A.carelessB.lawlessC.pointlessD.helpless4.A.reasonB.reminderC.compromise0.proposal5.A.informationB.interferenc

196、eC.entertainment0.equivalent6.A.byB.intoC.fromD.over7.A.linkedB. directedCpared8.A.dismissB.discoverC.create0.improve9.A.recallB.suggestC.select0.realize10.A.relcasedB.issuedC.distributed0.delivered11.A.carry onBdinger onC.set inD.log in12.A.ln vainB.ln effectC.ln returnD.ln contrast13.A.trustedBpet

197、ing14.A.cautionB.delightC.confidenceD.patience15.A.onB.afterC.beyond0.across16.A.dividedB.disappointedC. protected0.united17.A.frequestlyB.incidentally.occasionally0.eventually18.A.skepticismB.releranceC.indifference0.enthusiasm19. A.manageable B.defendable C.vulnerable D.invisible20. A.invited B.ap

198、pointed C.allowed D.forced完形填空参考答案1 5ACBDD 610BACCB 1115 DBACA 1620ADACDSection II Reading ComprehensionPart ADirections:Read the following four texts. Answer the questions after each text by choosing A, B, C orD. Mark your answers on ANSWER SHEET 1. (40points)Text 1Ruth Simmons joined Goldman Sachs

199、s board as an outside director in January 2000:a year later she became president of Brown University. For the rest of the decade sheapparently managed both roles without attracting much eroticism. But by the end of 2009Ms. Simmons was under fire for having sat on Goldmans compensation committee; how

200、could she have let those enormous bonus payouts pass unremarked? By February thenext year Ms. Simmons had left the board. The position was just taking up too much time,she said.Outside directors are supposed to serve as helpful, yet less biased, advisers on afirms board. Having made their wealth and

201、 their reputations elsewhere, they presumablyhave enough independence to disagree with the chief executives proposals. If the sky,and the share price is falling, outside directors should be able to give advice based onhaving weathered their own crises.The researchers from Ohio University used a data

202、base hat covered more than 10,000firms and more than 64,000 different directors between 1989 and 2004. Then they simplychecked which directors stayed from one proxy statement to the next. The most likelyreason for departing a board was age, so the researchers concentrated on those“surprise” disappea

203、rances by directors under the age of 70. They fount that after asurprise departure, the probability that the company will subsequently have to restateearnings increased by nearly 20%. The likelihood of being named in a federal class-actionlawsuit also increases, and the stock is likely to perform wo

204、rse. The effect tended to belarger for larger firms. Although a correlation between them leaving and subsequent badperformance at the firm is suggestive, it does not mean that such directors are alwaysjumping off a sinking ship. Often they “trade up. Leaving riskier, smaller firms for largerand more

205、 stable firms.But the researchers believe that outside directors have an easier time of avoiding ablow to their reputations if they leave a firm before bad news breaks, even if a review ofhistory shows they were on the board at the time any wrongdoing occurred. Firms whowant to keep their outside di

206、rectors through tough times may have to create incentives.Otherwise outside directors will follow the example of Ms. Simmons, once again verypopular on campus.21. According to Paragraph 1, Ms. Simmons was criticized for .Againing excessive profitsBfailing to fulfill her dutyCrefusing to make comprom

207、isesDleaving the board in tough times22. We learn from Paragraph 2 that outside directors are supposed to be .Agenerous investorsBunbiased executivesCshare price forecastersDindependent advisers23. According to the researchers from Ohio University after an outside directors surprisedeparture, the fi

208、rm is likely to .Abecome more stableBreport increased earningsCdo less well in the stock marketDperform worse in lawsuits24. It can be inferred from the last paragraph that outside d i r e c t o r s .Amay stay for the attractive offers from the firmBhave often had records of wrongdoings in the firmC

209、are accustomed to stress-free work in the firmDwill decline incentives from the firm25. The authors attitude toward the role of outside directors is .ApermissiveBpositiveCscornfulDcriticalT E X T 1 参考答案21. Ao细节题: 原文第1 段, 倒数第3 行 的 how could.?直接提到了 bonus payouts就是说 profits o22. Co 细节题: 原文中出现 outside d

210、irectors 有几处, helpful but less biased advisor,但 是 B 选项用的是executive,拼凑答案,D 选项也是一样。最后一句weatheredtheir own crises 对应 forecasterso23. C o 细节题:原文是若干个并列,stock is likely to perform worse对应答案, 迷惑选项是B , 但是主语不一致20%是 probability不是earningSo24. A o 推理题:原文对应firms who want t o 说想留住outside director就是增加incentiveo25.

211、 B o态度题:文章各个段落都说outside director的方面。因此是positive。Text 2Whatever happened to the death of newspaper? A year ago the end seemed near.The recession threatened to remove the advertising and readers that had not already fledto the internet. Newspapers like the San Francisco Chronicle were chronicling thei

212、r owndoom. Americas Federal Trade commission launched a round of talks about how to savenewspapers. Should they become charitable corporations? Should the state subsidizethem ? It will hold another meeting soon. But the discussions now seem out of date.In much of the world there is the sign of crisi

213、s. German and Brazilian papers haveshrugged off the recession. Even American newspapers, which inhabit the most troubledcome of the global industry, have not only survived but often returned to profit. Not the20% profit margins that were routine a few years ago, but profit all the same.It has not be

214、en much fun. Many papers stayed afloat by pushing journalistsoverboard. The American Society of News Editors reckons that 13,500 newsroom jobshave gone since 2007. Readers are paying more for slimmer products. Some papers evenhad the nerve to refuse delivery to distant suburbs. Yet these desperate m

215、easures haveproved the right ones and, sadly for many journalists, they can be pushed further.Newspapers are becoming more balanced businesses, with a healthier mix ofrevenues from readers and advertisers. American papers have long been highly unusualin their reliance on ads. Fully 87% of their reve

216、nues came from advertising in 2008,according to the Organization for Economic Cooperation & Development (OECD). InJapan the proportion is 35%. Not surprisingly, Japanese newspapers are much morestable.The whirlwind that swept through newsrooms harmed everybody, but much of thedamage has been concent

217、rated in areas where newspaper are least distinctive. Car andfilm reviewers have gone. So have science and general business reporters. Foreignbureaus have been savagely cut off. Newspapers are less complete as a result. Butcompleteness is no longer a virtue in the newspaper business.26. By saying “N

218、ewspapers like . their own doom” (Lines 3-4, Para. 1), the authorindicates that n e w s p a p e r .Aneglected the sign of crisisBfailed to get state subsidiesCwere not charitable corporationsDwere in a desperate situation27. Some newspapers refused delivery to distant suburbs probably b e c a u s e

219、.Areaders threatened to pay lessBnewspapers wanted to reduce costsCjournalists reported little about these areasDsubscribers complained about slimmer products28. Compared with their American counterparts, Japanese newspapers are much morestable because t hey.Ahave more sources of revenueBhave more b

220、alanced newsroomsCare less dependent on advertisingDare less affected by readership29. What can be inferred from the last paragraph about the current newspaper business?ADistinctiveness is an essential feature of newspapers.BCompleteness is to blame for the failure of newspaper.CForeign bureaus play

221、 a crucial role in the newspaper business.DReaders have lost their interest in car and film reviews.30. The most appropriate title for this text would be.AAmerican Newspapers: Struggling for SurvivalBAmerican Newspapers: Gone with the WindCAmerican Newspapers: A Thriving BusinessDAmerican Newspapers

222、: A Hopeless StoryTEXT 2 参考答案26. D o 定义题:根据上下文猜句子的含义,后 句 American. save newspaper中出现了 save说明前面的观点一定是不好的才s a v e ,因此选D。27. B .推理题:定位处前一句是 readers are paying more for slimmer newspaper.因此说明人们多付钱,报纸很薄,节约成本,定位处有even表示并列,说明前后的原因一致都是成本问题。28. C o推理题:日本美国原文用了对比的方法说广告占得比例不一样,因此问题是广告收入来源。29. D o 推理题:A 选项中有ess

223、ential,文章中是说distinctiveness重要而非必要,有问题,D 选项是文章中cars and film reviewers have gone, 说明由于报纸没有吸引力而失去读者。30. A o 主旨题:文章分析美国报纸出现的问题,说明要挽救。Text 3We tend to think of the decades immediately following World War II as a time ofprosperity and growth, with soldiers returning home by the millions, going off to col

224、lege onthe G. I. Bill and lining up at the marriage bureaus.But when it came to their houses, it was a time of common sense and a belief thatless could truly be more. During the Depression and the war, Americans had learned tolive with less, and that restraint, in combination with the postwar confid

225、ence in the future,made small, efficient housing positively stylish.Economic condition was only a stimulus for the trend toward efficient living. Thephrase “less is more was actually first popularized by a German, the architect LudwigMies van der Rohe, who like other people associated with the Bauha

226、us, a school ofdesign, emigrated to the United States before World War IIand took up posts at American architecture schools. These designers came to exertenormous influence on the course of American architecture, but none more so that Mies.Miess signature phrase means that less decoration, properly

227、organized, has moreimpact that a lot. Elegance, he believed, did not derive from abundance. Like othermodern architects, he employed metal, glass and laminated wood-materials that we takefor granted today buy that in the 1940s symbolized the future. Miess sophisticatedpresentation masked the fact th

228、at the spaces he designed were small and efficient, ratherthan big and often empty.The apartments in the elegant towers Mies built on Chicagos Lake Shore Drive, forexample, were smaller-two-bedroom units under 1,000 square feet-than those in theirolder neighbors along the citys Gold Coast. But they

229、were popular because of their airyglass walls, the views they afforded and the elegance of the buildings5 details andproportions, the architectural equivalent of the abstract art so popular at the time.The trend toward “less” was not entirely foreign. In the 1930s Frank Lloyd Wrightstarted building

230、more modest and efficient houses-usually around 1,200 square feet-thanthe spreading two-story ones he had designed in the 1890s and the early 20th century.The “Case Study Houses“ commissioned from talented modern architects byCalifornia Arts & Architecture magazine between 1945 and 1962 were yet ano

231、therhomegrown influence on the “less is more” trend. Aesthetic effect came from thelandscape, new materials and forthright detailing. In his Case Study House, Ralpheveryday life - few American families acquired helicopters, though most eventually gotclothes dryers - but his belief that self-sufficie

232、ncy was both desirable and inevitable waswidely shared.31. The postwar American housing style largely reflected the A m e r i c a n s * .Aprosperity and growthBefficiency and practicalityCrestraint and confidenceDpride and faithfulness32. Which of the following can be inferred from Paragraph 3 about

233、 Bauhaus?AIt was founded by Ludwig Mies van der Rohe.Blts designing concept was affected by World War II.CMost American architects used to be associated with it.Dlt had a great influence upon American architecture.33. Mies held that elegance of architectural desi gn.Awas related to large spaceBwas i

234、dentified with emptinessCwas not reliant on abundant decorationDwas not associated with efficiency34. What is true about the apartments Mies building Chicagos Lake Shore Drive?AThey ignored details and proportions.BThey were built with materials popular at that time.CThey were more spacious than nei

235、ghboring buildings.DThey shared some characteristics of abstract art.35. What can we learn about the design of the “Case Study House”?AMechanical devices were widely used.BNatural scenes were taken into considerationCDetails were sacrificed for the overall effect.DEco-friendly materials were employe

236、d.TEXT 3 参考答案31. Co 细节题:原文 restraint, in combination with the postwar confidence 对应。32. D o 推理题: 定位是Bauhaus,对应选项与原文,只有D 对。33. Co 细节题:原文 elegance did not derive from abundance。34. Do 细节题:原文 But 后有 the architectural equivalent of the abstract art 35. B J 隹理题: 原文 Aesthetic effect came form the landscap

237、e, new materials and forthrightdetailing0Text 4Will the European Union make it? The question would have sounded strange not longago. Now even the projecfs greatest cheerleaders talk of a continent facing a Bermudatriangle” of debt, population decline and lower growth.As well as those chronic problem

238、s, the EU face an acute crisis in its economic core,the 16 countries that use the single currency. Markets have lost faith that the euro zoneseconomies, weaker or stronger, will one day converge thanks to the discipline of sharing asingle currency, which denies uncompetitive members the quick fix of

239、 devaluation.Yet the debate about how to save Europes single currency from disintegration isstuck. It is stuck because the euro zones dominant powers, France and Germany, agreeon the need for greater harmonization within the euro zone, but disagree about what toharmonies.Germany thinks the euro must

240、 be saved by stricter rules on borrow spending andcompetitiveness, barked by quasi-automatic sanctions for governments that do not obey.These might include threats to freeze EU funds for poorer regions and EU mega-projectsand even the suspension of a countr/s voting rights in EU ministerial councils

241、. It insiststhat economic co-ordination should involve all 27 members of the EU club, among whomthere is a small majority for free-market liberalism and economic rigour; in the inner corealone, Germany fears, a small majority favour French interference.A “southern“ camp headed by French wants someth

242、ing different: Europeaneconomic governmenf within an inner core of euro-zone members. Translated, thatmeans politicians intervening in monetary policy and a system of redistribution from richerto poorer members, via cheaper borrowing for governments through common Eurobondsor complete fiscal transfe

243、rs. Finally, figures close to the France government havemurmured, euro-zone members should agree to some fiscal and social harmonization:e.g., curbing competition in corporate-tax rates or labour costs.It is too soon to write off the EU. It remains the worlds largest trading block. At itsbest, the E

244、uropean project is remarkably liberal: built around a single market of 27 richand poor countries, its internal borders are far more open to goods, capital and labourthan any comparable trading area. It is an ambitious attempt to blunt the sharpest edgesof globalization, and make capitalism benign.36

245、. The EU is faced with so many problems that.A it has more or less lost faith in marketsB even its supporters begin to feel concernedC some of its member countries plan to abandon euroD it intends to deny the possibility of devaluation37. The debate over the EUs single currency is stuck because the

246、dominantpower s .A are competing for the leading positionB are busy handling their own crisesC fail to reach an agreement on harmonizationD disagree on the steps towards disintegration38. To solve the euro problem .Germany proposed t hat .A EU funds for poor regions be increasedB stricter regulation

247、s be imposedC only core members be involved in economic co-ordinationD voting rights of the EU members be guaranteed39. The French proposal of handling the crisis implies t hat .A poor countries are more likely to get fundsB strict monetary policy will be applied to poor countriesC loans w川 be readi

248、ly available to rich countriesD rich countries will basically control Eurobonds40. Regarding the future of the EU, the author seems to f eel .A pessimisticB desperateC conceitedD hopefulTEXT 4 参考答案36. Bo 推理题:第一段 But 后说 cheerleader 觉得 EU 有 debt, decline lower growth o37. D o 推理题:三段论德法对欧元区和谐上达成致但如何和谐有

249、分歧。38. Bo 细节题:原文对应 by stricter rules on。39. Ao 推理题: 原文对应 a system of redistribution from richer to poorer members, viacheaper borrowing for governments through common Eurobonds.40. D e 态度题:文章最后总结认为EU是 worlds largest trading block.最后一句it is anambitious attempt to blunt the sharpest edges of globaliza

250、tion, and make capitalismbenign都是说EU正面的信息。Part BDirections:Read the following text and answer the questions by finding information from the rightcolumn that corresponds to each of the marked details given in the left column. There aretwo extra choices in the right column. Mark your answer on ANSWER

251、SHEET 1. (10points)Such a move could affect firms such as McDonalds, which sponsors the youthcoaching scheme run by the Football Association. Fast-food chains should also stopoffering “inducements” such as toys, cute animals and mobile phone credit to lure youngcustomers, Stephenson said.Professor D

252、inesh Bhugra, president of the Royal College of Psychiatrists, said: Ifchildren are taught about the impact that food has on their growth, and that some thingscan harm, at least information is available up front.”He also urged councils to impose ufast-food-free zones” around school andhospitals-area

253、s within which takeaways cannot open.A Department of Health spokesperson said: uWe need to create a new vision for publichealth where all of society works together to get healthy and live longer. This includescreating a new responsibility deaf with business, built on social responsibility, not state

254、regulation. Later this year, we will publish a white paper setting out exactly how we willachieve this.”The food industry will be alarmed that such senior doctors back such radical moves,especially the call to use some of the tough tactics that have been deployed againstsmoking over the last decade.

255、A fat taxes should be imposed on fast-foodproducers such as McDonalds.41.Andrew Lansley held thatB the government should ban fast-foodoutlets in the neighborhood of schools.42.Terence Stephenson agreedthatC lecturing was an effective way to improveschool lunches in England.43.Jamie Oliver seemed tob

256、elieve thatD cigarette-style warnings should beintroduced to children about the dangers of apoor diet.44.Dinesh Bhugra suggested thatE the producers of crisps and candies couldcontribute significantly to theChange4Life campaign.45.A Department of HealthSpokesperson propsed thatF parents should set g

257、ood examples for theirchildren by keeping a healthy dietat home.G the government should strengthen thesense of responsibility among businesses.Part B参考答案41. E o 原文第6 段第2 行:manufacturer of crisps. Play acentral role in the Change4life.42. D e 原文第7 段全部。43. B o 原文第6 段第5 行 Jamie Oliver处。44. Bo 原文第 10 段:

258、imposefast food free zone。45. G o 原文第11段全部。46. Direction:In this section there is a text in English. Translate it into Chinese, write your translation onANSWER SHEET 2. (15points)Who would have thought that, globally, the IT industry produces about the samevolumes of greenhouse gases as the worlds a

259、irlines do-rough 2 percent of all CO2emissions?Many everyday tasks take a surprising toll on the environment. A Google search canleak between 0.2 and 7.0 grams of CO2 depending on how many attempts are needed toget the “right“ answer. To deliver results to its users quickly, then, Google has to main

260、tainvast data centres round the world, packed with powerful computers. While producing largequantities of CO2, these computers emit a great deal of heat, so the centres need to bewell air-conditioned, which uses even more energy.However, Google and other big tech providers monitor their efficiency c

261、losely and makeimprovements. Monitoring is the first step on the road to reduction, but there is much to bedone, and not just by big companies.参考答案从全球范围来看,有谁会想到IT 行业释放的温室气体与全球航空公司产生的一样多呢?它大约占总二氧化碳总排量的2%。许多日常工作对环境造成了令人震惊的破坏。 根据每次你搜索并得到正确答案的尝试次数,谷歌会排放0.2至 7 克的二氧化碳。为了迅速将结果传递给用户,谷歌在全球设置了大量充斥着能量巨大的电脑的数据中

262、心。 这些电脑在排放大量二氧化碳的同时, 也产生大量的能量。因此,这些数据中心需要良好的空调降温,这又会同时产生大量的能量。然而, 谷歌和其他技术提供商严密检测他们的效果并不断进行改进。监控是减排的第一步,但这仍任重道远,且不仅只山大公司来承担。Section IV WritingPart A47 Directions:Suppose your cousin Li Ming has just been admitted to a university. Write him/her aletter to1) congratulate him/her, and2) give him/her sug

263、gestions on how to get prepared for university life.You should write about 100 words on ANSWER SHEET 2.Do not sign your own name at the end of the letter. Use Zhang Wer instead.Do not write the address. (10 points)Part B48 write a short essay baesd on the following chart.in your writing,you should:1

264、)interpret the chart and2)give your commentsyou should write at least 150 wrodswrite your essay on answer sheet 2(15points)2008年 2009年口国产品牌 日系品牌 美系品牌2008、2009年国内轿车市场部分品牌市场份额示意图小作文参考答案范文一Dear Li Ming,We are very happy to know that you have successfully passed the college entranceexamination this year

265、 and have been admitted into Peking University. Allow us to give ourmost sincere congratulations on this exciting occasion.You have all along been working hard at your professional studies, and you areexcellent in most subjects. Your success shows that only hard work can yield goodresults, so I sugg

266、est that you should make a great progress in university life.We take this opportunity to express our best wishes to you. Wish you greaterachievements in your college education.Yours sincerely,Zhang Wei范文二Dear Ming,Congratulations! I am glad to hear that you have been admitted by MIT. Your efforts an

267、dcommitment have been paid off. You are the honor of our family.Here come some my own advices of being a pre college student. First and foremost,you need to improve your communication because you will meet different people withdifferent personalities in campus. Moreover, reading some reference books

268、 will help youto accumulate more knowledge and terms, which boost your competitiveness in campus.Once again congratulate for your achievement!Yours sincerely,Zhang Wei范文三:Dear Liming,Congraduate on you success in passing the entrance examination.Now, please allow me to give you some suggestion durin

269、g your holidays.First of all, you should read. Because it makes a full man. Reading during the holidayhelps you get the habit of it that when you become a freshman. College life is so plentifulbut reading is the most important thing.Second, to do some housework can bring you another feelings. Once y

270、ou get into thecollege, you must do the things for yourself, including washing, clear the room andshedule your daily life and etc.However, reading and housework doesnt mean all of your holiday. You need contactwith your friends or communicate with them. The reason is that old friends will be in your

271、memory and new friends will be there. And we all know that the friendship among seniorschool.From the things I mentioned above, hope they will bring you a richful life in your college.Zhang wei大作文参考答案As is shown in the bar chart above, dramatic changes have taken place in the autosmarket shares with

272、in two years (from 2008 to 2009). The most obvious change was themarket share of national brand, which had increased nearly by 10%, while Japans autosmarket share decreased roughly by 10%. The percentage of the US autos remainedstable between 2008 and 2009.There are numerous reasons accounting for t

273、he phenomenon and I would like toexplore a few of the most important ones here. Above all, as the development oftechnique and knowledge in native companies, a growing number of autos corporationdeveloped many quality autos. Therefore, the national people changed the attitude to thenative brands and

274、acknowledge them. Whaf more, an overwhelming majority of peoplehave been affected by the country patriotism ideology, partly owing to some actions ofJapan triggering the emotion of people. Finally, Toyota brake error accidents significantlyaffects Japanese autos* reputations and images. Safety conce

275、rns drove customers awayfrom Japanese products. Additionally, Fuel price drove consumers away from thoseAmerican petrol digging and luxury autos. So it is not difficult to observe their steadyperformance.Based on what has been discussed above, we may reasonably conclude that thetendency described in

276、 graphic will continue for quite a long time. Hopefully, governmentcould offer more friendly policies to China autos manufacturers to encourage qualityimprovement and technology innovation.( 五 ) 2010年 MBA综合真题及答案一. 问题求解:第 1-15小题,每小题3 分,共 45 分,下列每题给出的A、B、C、D、E五个选项中,只有一项是符合试题要求的,请在答题卡上将所选项的字母涂黑1 .电影开演时

277、观众中女士与男士人数之比为5: 4 , 开演后无观众入场,放映一个小时后,女士的2 0 % ,男士的15%离场,则此时在场的女士与男士人数之比为(A) 4: 5 (B) 1: 1 (C) 5: 4 (D) 20: 17 (E) 85: 642 . 某商品的成本为240元,若按该商品标价的8 折出售,利润率是1 5 % ,则该商品的标价为(A) 276 元 (B)331 元 9 ) 345 元(D) 360 元 (E) 400 元3 . 三名小孩中有一名学龄前儿童( 年龄不足6 岁) ,他们的年龄都是质数( 素数) ,且依次相差 6 岁,他们的年龄之和为(A) 21 (B) 27 (C) 33

278、(D) 39 (E) 514 . 在右边的表格中每行为等差数列,每列为等比数列,x+y+z=(A) 2 ( B ) -27(D) - (E) 42(C) 32523X5432ay34bcz5 . 如图1.在直角三角形ABC区域内部有座山,现计划从BC边上某点D 开凿一条隧道到点A , 要求隧道长度最短,一直A B 长为5km, A C 长 为 12km,则所开凿的的隧道A D 的长度约为(A) 4.12km (B) 4.22km (C) 4.42km (D) 4.62km (E) 4.92km6 . 某商店举行店庆活动,顾客消费达到一定数量后,可以在4 中赠品中随即选取2 个不同的赠品,任意两

279、位顾客所选赠品中,恰 有 1 件品种相同的概率是(A) - (B) - (C) - (D) - ( E ) -6 4 3 2 37 . 多项式 x3+x2+ fe x -6 的两个因式是x-1和 x -2 ,则第三个一次因式为(A) x-6 (B) x-3 (C) x+1 (D) x+2 (E) x+38 . 某公司的员工中, 拥有本科毕业证,计算机登记证, 汽车驾驶证得的人数分别为130.110.9 0 ,又知只有一种证的人数为1 4 0 ,三证齐全的人数为3 0 ,则恰有双证的人数为(A) 45 (B) 50 (C) 52 (D) 65 (E) 1009 . 甲商品销售某种商品, 该商品的

280、进价每件90元, 若每件定位100元, 则一天内能售出500件,在此基础上,定价每增1 元, 天能使少售出10件,甲商店获得最大利润,则该商品的定价应为(A) 115 元(B) 120 元(C) 125 元 (D) 130 元 (E) 135 元10.em aax-by+3=0 (a0,b0) i l l i x2+4x + y2- 2 y + 1 = 0 的圆心,则 a-b 的最大值为11 . 某大学排除5 名志愿者到西部4 所中学指支教,若每所中学至少有名志愿者,则不同的分贝方案共有(A) 240 种(B) 144 种 (C) 120 种 (D )60 种 (E )24 种12 . 某装置

281、的启动密码是由0 到 9 中的3 各不同数字组成,连续3 次输入错误密码,就会导致该装置永久关闭,一个仅记得密码是由3 个不同数字组成的人能够启动此装置的概率为( A )击(B) ( C ) 168 240( D ) 72013. 某居民小区决定投资15万元修建停车位,据测算,修建一个室内的费用为5000元,修建一个室外车位的费用为1000元,考虑到实际因素,计划室外车位的数量不少于室内车位的 2 倍,也不多于室内车位的3 倍,这笔投资最多可见车位的数量为(A) 78 (B) 74 (C) 72 (D) 70 (E) 661 4 ,如图2 , 长方形ABCD的两天边分别为8m 和 6 m ,四

282、边形OEFG的面积是4/,则阴影部分的面积为(A) 32 m2 (B)28 m2 (C) 24 m2 (D)20m2 (E)16/215. 再一次竞猜活动中,设有5 关,如果连续通过2 关就算闯关成功,小王通过每关的概率都 是 他 闯 关 成 功 得 该 率 为21 1 3 4 19(A) - (B) - (C) - (D) - (E)8 4 8 8 32二、 条件充分性判断: 第 16-25小题, 每小题3 分, 共 30分。 要求判断每题给出得条件(1)和(2 ) 能否充分支持题干所陈述的结论。A.B.C.D.E.五个选项为判断结果,请选择一项符合试题要求得判断,在答题卡上将所选项得字母涂

283、黑。( A ) 条 件(1 ) 充分,单但条件(2 ) 不充分(B ) 条 件(2 ) 充分,单但条件( 1 ) 不充分(C ) 条 件 ( 1 ) 和 条 件 ( 2 ) 单独都不充分,但 条 件 ( 1 ) 和 条 件 ( 2 ) 联合起来充分(D ) 条 件 ( 1 ) 充分,单但条件( 2 ) 不也分(E ) 条 件(1 ) 和 条 件 ( 2 ) 单独都不充分,条 件 ( 1 ) 和 条 件(2 ) 联合起来也不充分16.a|a-Z?|a|(a-Z?) 实 数 a0(2)实数a,b满足ab17 . 有偶数位来宾(1)聚会时所有来宾都被安排坐在一张圆桌周围,且每位来宾与其邻座性别不同。

284、(2) 聚会时男宾人数是女宾人数的两倍。18 . 售出件甲商品比售出一件乙商品利润要氤(1 ) 售出5 件甲商品,4 件乙商品共获利50元。( 2 ) 售出4 件甲商品,5 件乙商品共获利47元。19. 已知数列 4 为等差数列,公差为d, 4 + 4 + / + % = 1 2 , 贝 I =0(1) d=-2(2) a2 + % = 420 . 甲企业今年人均成本是去年的60%。( 1 ) 甲企业今年总成本比去年减少2 5 % ,员工人数增加25%。(2 ) 甲企业今年总成本比去年减少2 8 % ,员工人数增加20%。21 . 该股票涨了( 1 ) 某股票连续三天涨10%后,又连续三天跌1

285、0%。(2 ) 某股票连续三天跌10%后,又连续三天涨10%。22 . 某班有5 0 名学生,其中女生2 6 名,一直在某次选拔测试中,有 27 名学生未通过,则有 9 名男生通过。( 1 ) 在通过的男生中,女生比男生多5 人。(2 ) 在男生中未通过的人数比通过的人数多6 人。23 . 甲企业一年的总产值为- ( 1 + p产一1(1)甲企业一月份的产值为a , 以后每月产值的增长率为p(2 ) 甲企业一月份的产值为以后每月产值的增长率为2P24 . 设a,b为非负实数,则4( 1) ah 16(2) a2+b2 /2BF26 . 针对威胁人类健康的甲型H1N1流感,研究人员研制出了相应的

286、疫苗,尽管这些疫苗是有效的, 但某大学研究人员发现,阿司匹林, 羟苯基乙酰胺等抑制某些酶的药物会影响疫苗的效果,这位研究员指出:“ 如果你服用了阿司匹林或者乙酰基酚,那么你注射疫苗后就必然不会产生良好的抗体反映。”如果小张注射疫苗后产生了良好的抗体反映,那么根据上述研究结果可以得出一下哪些结论?(A ) 小张服用了阿司匹林,但没有服用对乙酰基酚。(B ) 小张服没有用阿司匹林,但感染了 H1N1流感病毒。( C)小张服用了阿司匹林,但没有感染了 H1N1流感病毒。(D)小张服没有用了阿司匹林,也没有服用对乙酰基酚。(E)小张服用了乙酰基酚,但没有服用羟苯基乙酰胺。27. 为了调查但当前的人们的

287、识字水平,其饰演者列举了 20个词语,请 30位文化识读,这些人的文化程度都在大专以上。识读结果显示,多数人只读对3 到 5 个词语,极少数人读对 15以上,甚至有人全部读错。其中,“ 蹒跚” 的辨识率最高,30人中有19人读对,“ 呱呱坠地” 所有人都读错, 20 个词语的整体误读率接近80% ,该试验者由此得出,当前人们的识字水平并没有提高,甚至有所下降。以下哪项如果是真,最能对该实验者的结论构成质疑?(A )实验者选取的20个词语不具有代表性。(B)实验者选取的30位识读者均没有博士学位。(C)实验者选取的20个词语在网络流行语言中不常用。(D) “ 过过坠地” 这个词的读音有些大学老师

288、也经常读错。(E)实验者选取的30位识读者约有50%人学成绩不佳。28 . 域控制器储存了域内的账户,密码和属于这个城市的计算机三项信息。当计算机接入网络时,域控制器首先要鉴别这台计算机是否属于这个域,用户使用的登陆账户是否存在,密码是否正确,如果三项信息均是正确,则允许登陆;如果以上信息有一项不正确,那么域控制器就会拒绝这个用户从这台计算机登陆。 小张的登陆帐号是正确的, 但是域控制器拒绝小张的计算机登陆。基于以上陈述能得到一下哪些结论?(A )小张输入的密码是错误的。(B)小张的计算机不属于这个域。(C)如果小张输入的密码是正确的,他的计算机才属于这个域。(D)只有小张输入的密码是正确的,

289、它的计算机才属于这个域。(E)如果小张输入的密码是iE确的,那么他的计算机属于这个域。29 . 现在越来越多的人拥有了自己的轿车,但他们明显地缺乏汽车保养的基本知识。这些人会按照维修保养手册或4 s 店后的售后服务人员的提示做定期保养。可是,某位有经验的司机会告诉你,每 行 驶 5 千米公里做一次定期检查,只能检查出汽车可能存在问题的一小部分,这样的检查是没有意义的,是浪费时间和金钱。(A) 每行驶5 千米公里做一次定期检查是保障车主安全所需要的。(B) 每行驶5 千米公里做一次定期检查能发现引擎的某些主要故障。( C) 在定期检查中所做的常规维护是保证汽车正常运行所必须的。(D) 赵先生的新

290、车作定期检查行驶到5100公里时出了问题。( E) 某公司新购的一批汽车未作定期检查,均安全行驶了 7000公里以上。30 . 化学课上,张老师演示了两个同时进行得教学实验:一个实验是KCIO3加热后0 2 缓慢产生:另一个实验KCIO3加热后迅速撒入少量M nO 2,这时立即有大量的0 2 产生。张老师山此指出: MNO2是 0 2 快速产生得原因。以下哪项与张老师得出结论得方法类似?(A)同一品牌的化妆品价格越高卖得越火。由此可见,消费者喜欢价格高得化妆品。(B) 居里夫人在沥青矿物中提取放射性元素时发现, 从一定量的沥青矿物中提取的全部纯铀得放射线强度比同等数量的沥青矿物质中放射线迁都低

291、数倍。 她据此推断, 沥青矿物中还存在其它放射性更强元素。(C)统计分析发现,3 0 岁-6 0 岁之间,年纪越大胆子越小,有理由相信;岁月是勇敢的腐蚀剂。(D)将闹钟放在玻璃罩里,使它打铃,可以听到铃声; 然后把玻璃罩里得空气抽空,再使闹钟打铃,就听不到铃声了。由此可见,空气是声音得传播介质。(E)人们通过对绿藻、 蓝藻、 红藻得大量观察, 发现结构简单. 、 无根叶是植物得的主要特征。31 . 湖队是不可能进入决赛的。如果湖队进入决赛,那么太阳就从西边出来了。以下哪项与上述论证方式最相似?(A)今天天气不冷。如果冷,湖面怎么结冰了?(B)语言是不能创造财脆。其死也枯槁。故坚强者也死之徙,柔

292、弱着生之徙。(D)天上是不会掉馅饼得。若果你不相信这一点,那上当受骗是迟早的事。(E)古典音乐不流行。若果流行,那就说明大众的音乐欣赏水平打大大提高了。32 . 在某次课程教学改革研讨会上,负责工程类教学的程老师说,在工程设计中,用于解决数学问题的计算机程序越来越多了, 这样就不必要求工程技术类大学生对基础数学有深刻的理解。因此,在未来的教学中,基础数学课程可以用其它重要的工程类课程代替。以下哪项若果为真,能削弱程老师得上述论证?I , 工程类基础课程中已经包含了相关得基础数学的内容。I I , 在工程设计中,设计计算机程序需要对基础数学有全面的理解。I II,基础数学课程的一 个 重要目标是

293、培养学生得思维能力,这种能力对工程设计来说很关键。( A ) 只有II (B)只有I 和 II和川(D)只有II和川(E) I、II 和 III33 . 蟋蟀是一种非常有趣的小动物,宁静的夏夜,草从中传来阵阵清脆悦耳得鸣叫声,那是蟋蟀在唱歌, 蟋蟀优美动听的歌声并不是出自它的好嗓子, 而是来自它得翅膀。左右两翅一张一合,相互摩擦,就可以发出悦耳的声响了。蟋蟀还是建筑专家。与它那柔软的挖掘工具相比,蟋蟀的住宅门口,有一个收拾得非常舒适得平台,夏夜,除非下雨或者刮风,否则蟋蟀肯定会在这个平台上唱歌。根据以上陈述,以下哪项是蟋蟀在无雨的夏夜所做的?(A)修建住宅。(B)收拾平台。(C)如果没有刮风,

294、它就在抢修工程。(D)如果没有刮风,它就在平台上唱歌。34 . 一般认为,出生地间隔较远的夫妻所生子女的智商较高。有资料显示,夫妻均是本地人,所生子女的平均智商为102.45;夫妻是省内异地的,其所生子女的平均智商为106.17;而隔省婚配的,其所生子女的智商刚高达109.35。因此,异地通婚可提高下一代智商水平。以下哪项如果为真,最能削弱上述结论?(A)统计孩子平均智商的样本数量不够多。(B)不难发现, 些天才儿童的父母的均是本地人(C)不难发现,一些低智商儿童父母的出生地间隔较远。(D)能够异地通婚者是智商比较高的,他们自身的高智商促成了异地通婚。(E)一些情况下,夫妻双方出生地间隔很远,

295、但他们的基因可能接近。35 . 成品油生产商的利润很大程度上受国际市场原油价格的影响,因为大部分原油是按国际市场价购进的。近年来,随着国际原油市场市场价格的不断提高,成品油生产商的运营成本大幅度增加,但某国成品油生产商的利润并没有减少,反而增加了。以卜哪项如果为真,最有助于解释上述看似矛盾的现象?(A)原油成本只占成品油生产商运营成本的一半。(B)该国成品油价格根据市场供需确定。随着国际原油市场价格的上涨,该国政府为成品油生产商提供相应的补贴。(C)在国际原油市场价格不断上涨期间,该国成品油生产商降低了个别高薪雇员的工资。(D)在国际原油市场价格上涨之后,除进口成本增加以外,成品油生产的其他运

296、营成本也有所提高。(E)该国成品油生产商的原油有一部分来自国内,这部分受国际市场价格波动影响较小。36 . 太阳风中的一部分带电粒子可以到过M 星表面,将足够的能量传递给M 星表面粒子,使后者脱离M 星表面,逃逸到M 星大气中。为了判定这些逃逸的粒子,科学家们通过三个实验获得了如下信息:实验一:或者是X 粒子,或者是丫粒子;实验二:或者不是丫粒子,或者不是Z 粒子;实验三:如果不是Z 粒子,就不是丫粒子。根据上述三个补给, 以下哪项一定为真?(A)这种粒子是X 粒子。 (B)这种粒子是丫粒子。(C)这种粒子是Z 粒子。 (D)这种粒子不是X 粒子。(E)这种粒子不是Z 粒子。37 . 美国某大

297、学医学院的研究人员在 小儿科杂志 上发表论文指出,在 对 2702个家庭的孩子进行跟踪调查后发现, 如 果 孩 子 在 5 岁前每天看电视超过2 小时,他们长大后出现行为问题的风险将会增加1 倍多。所调行为问题是指性格孤僻,言行粗鲁,侵犯他人,难与他人合作等。(A ) 电视节目会使孩子产生好奇心,容易导致孩子出现暴力倾向。(B ) 电视节目中有不少内容容易使孩子长时间处于紧张、恐惧的状态。(C ) 看电视时间过长,会影响儿童与他人的交往,久而久之,孩子便会缺乏与他人打交道的经验。(D ) 儿童模仿力强,如果只对电视节目感兴趣,长此以往,会阻碍他们分析能力的发展。(E ) 每天长时间地看电视,容

298、易使孩子神经系统产生疲劳,影响身心健康发展。38 . 一种常见的现象是,从国外引进的一些畅销科普读物在国内并不畅销,有人对此解释说这与我们多年来沿袭的文理分科有关。 文理分科人为地造成了自认科学与人文社会科学的割裂,导致科普类图书的读者市场还没有真正形成。一下哪项如果成真,最能加强上述观点?(A ) 有些自然科学工作者对科普读物也不感兴趣。(B ) 科普读物不是没有需求, 而是有效供给不足。(C)由于缺乏理科背景,非自然科学工作者对科学敬而远之。(D)许多科普电视节目都拥有固定的收视群,相应的科普读物也就大受欢迎。( E)国内大部分科普读物只是介绍科学知识,很少真正关注科学精神的传播。39 .

299、 大小行星悬浮游在太阳系边缘,极易受附近星体引力作用的影响。据研究人员计算,有时这些力量会将彗星从奥尔特星云拖出。这样,它们更有可能靠近太阳。两位研究人员据此分别作出了以下两种有所不同的断定:一、木星的引力作用要么将它们推至更小的轨道,要么将它们逐出太阳系;二、木星的引力作用或者将它们推至更小的轨道, 或者将它们逐出太阳系。如果上述两种断定只有一种为真,可以退出以下哪项结论?(A )木星的引力作用将它们推至更小的轨道,并且将它们逐出太阳系。(B)木星的引力作用没有将它们推至更小的轨道,但是将它们逐出太阳系。(C)木星的引力作用将它们推至更小的轨道,但是没有将它们逐出太阳系。(D)木星的引力作用

300、既没有将它们推至更小的轨道,也没有将它们逐出太阳系。(E)木星的引力作用如果将它们推至更小的轨道,就不会将它们逐出太阳系。40 . 鸽子走路时,头部并不是有规律地前后移动,而是一直在往前伸。行走时,鸽子脖子往前一探, 然后头部保持静止,等待着身体和爪子跟进。 有学者曾就鸽子走路时伸脖子的现象作出假设:在等待身体跟进的时候, 暂时静止的头部有利于鸽子获得稳定的视野, 看清周围的食物。以下哪项如果为真,最能支持上述假设?(A )鸽子行走时如果不伸脖子,很难发现远处的食物。(B)步伐杜宇鸟类,伸缩脖子的幅度远比步伐小的要大。(C)鸽子行走速度的变化,刺激内耳控制平衡的器官,导致伸脖子。(D)鸽子行走

301、时一举翅一投足,都可能出现脖子和头部肌肉的自然反射,所以头部不断运动。(E)如果雏鸽步态受到限制,功能发育不够完善,那么,成年后鸽子的步伐变小,脖子伸缩幅度则会随之降低。41.S市环保监测中心的统计分析表明,2009年空气质量为优的天数达到了 150天, 比 2008年多出22天:二氧化碳、一氧化碳、二氧化氮、可吸入颗粒物四项污染物浓度平均值,与2008年相比分另IJ下降了约21.3%、25.6%、26.2%、15.4%, S 市环保负责人指出,这得益于近年来本市政府持续采取的控制大气污染的相关措施。以卜除哪项外,均能支持上述S 市环保负责人的看法?( A) S 市广泛开展环保宣传,加强了市民

302、的生态理念和环保意识。( B) S 市启动了内部控制污染方案;凡是排放不达标的燃煤锅炉停止运行。( C) S 市执行了机动车排放国IV标准,单车排放比III标准降低了 49%o( D) S 市市长办公室最近研究了焚烧秸秆的问题,并着手制定相关条例。(E) S 后制定了“ 绿色企业” 标准,继续加快污染重、能耗高企业的退出。42. 在某次思维训练课上,张老师提出“ 尚左数” 这一概念的定义;在连续排列的一组数字中,如果一个数字左边的数字都比其大( 或无数字) ,且其右边的数字都比其小( 或无数字) ,则称这个数字为尚左数。根据张老师的定义,在 8, 9, 7, 6, 4, 5, 3, 2 这列数

303、字中,以下哪项包含了该列数字中所有的尚左数?( A ) 4、5、7 和 9( B ) 2、3、6 和 7(C) 3、6、7 和 8( D) 5、6、7 和 8(E) 2、3、6 和 843 . 一般认为,剑乳齿象是从北美洲迁入南美洲的。剑乳齿象的显著特征是具有较真的长剑形门齿,鄂骨较短,臼齿的齿冠隆起,齿板数目为7至8个,并呈乳状凸起,剑乳齿象因此得名,剑乳齿象的牙齿结构比较复杂,这表明它能吃草。在南美洲的许多地方都有证据显示史前人类捕捉过剑乳齿象。由此可以推测, 剑乳齿象的灭绝可能与人类的过度捕杀有密切关系。以下哪项如果为真,最能反驳上述论证?(A )史前动物之间经常发生大规模相互捕杀的现象

304、。(B )剑乳齿象在遇到人类攻击时缺乏自我保护能力。(C)剑乳齿象也存在由南美洲进入北美洲的回迁现象。(0 )由于人类活动范围的扩大,大型食草动物难以生存。(E)幼年剑乳齿象的牙齿结构比较简单, 自我生存能力弱。44 . 小东在玩, 勇士大战,游戏,进入第二关时,界面出现四个选项,第 个 选 项 是 “ 选择任意选项都需要支付游戏币” , 第二个选项是“ 选择本项后可以得到额外游戏奖励” , 第三个选项是“ 选择本项游戏后游戏不会进行下去” ,第四个选项是“ 选择某个选项不需支付游戏币。如果四个选项中的陈述中有一名为真,则以下哪项一定为真?(A )选择任意选项都需支付游戏币。(B)选择任意选项

305、都无需支付游戏币。(C)选择任意选项都不能得到额外游戏奖励(D)选择第二个选项后可以得到额外游戏奖励( E)选择第三个选项后游戏能继续进行下去。45 . 有们美国学者做了一个实验,给被试儿童看三辐图画,鸡牛青草,然后让儿童将其分为两类。结果大总分中国儿童把牛和青草一类,把鸡归为另一类,大部分美国儿童则把牛和鸡归为一类,把青草归为另一类。这们美国学者由此得出:中国儿童习惯于按照事物之间的关系来分类,美国儿童则习惯于把事物按照各自所属的“ 实体” 范畴进行分类。以卜哪项是这门学者得出结论所必须假设的?A马和青草是按照副手之间的关系被列为一类。B鸭和鸡蛋是按照各自所属的 实体 范畴被归为一类。C美国

306、儿童只要把牛和鸡归为一类,就是习惯于按照各自所属 实体 范畴进行分类。D美国儿童只要把牛和鸡归为一类,就不是习惯于按照事物之间的关系来分类。E中国儿童只要把牛和青草归为一类, 就不是习惯于按照各自所属 实体 范畴进行分类。46 . 相互尊重是相互理解的基础,相互理解是相互信任的前提;在人与人的相互交往中, 自重, 自信也是非常重要的,没有一个人尊重不自重的人。没有一个人信任他所不尊重的人。以上陈述可以推出以下结论?A不自重的人也不被任何人信任。B 相互信任才能相互尊重。C 不自信的人也不自重。D 不自信的人也不被任何人信任E 不自信的人也不受任何人尊重47学生: IQ和 EQ哪个更重要? 您能

307、否给我指点一下?学生:你去书店问问工作人员,关于IQ,EQ的书哪类销得快,哪类就更重要。以下哪项与上述题干中的问答方式最为相似?A 员工: 我们正制定一个度假方案, 你说是在本市好,还是去外地好?经理: 现在年终了, 个公司都在安排出去旅游, 你去问问其它公司的同行, 他们计划去哪里,我们就不去哪里,不凑热闹。B:平平:母亲节那天我准备给妈妈送一样礼物,你说是送花还是送巧克力好?佳佳:你在母亲节前一天去花店看一下,看看买花的人多不多就行了嘛.C:顾客:我准备买一件毛衣,你看颜色是鲜艳一点,还是素一点好?店员:这个需要结合自己的性格与穿衣习惯,各人可以有自己的选择与喜好。D:游客:我们前面有两条

308、山路,走哪一条更好?导游:你仔细看看,哪一条山路上车马的痕迹深, 我们就走哪一条。E 学生:我正在准备期末复习,是做教材上的练习重要还是理解教材内容更重要?老师:你去问问高年级得分高的同学,他们是否经常背书做练习。4 8 李赫,张岚,林宏,何柏,邱辉五位同事,近 II他们各自买了一辆不同品牌小轿车,分别为雪铁龙,奥迪,宝马,奔驰,桑塔纳。这五辆车的颜色分别与五人名字最后一个字谐音的颜色不同。已知李赫买的是蓝色的雪铁龙。以下哪项排列可能依次对应张岚,林宏, 何柏,邱辉所买的车?A 灰色的奥迪,B 黑色的奥迪,C 红色的奥迪,D 白色的奥迪,E 黑色的奥迪,白色的宝马,灰色的奔驰,红色的桑塔纳红色

309、的宝马,灰色的奔驰,白色的桑塔纳灰色的宝马,白色的奔驰,黑色的桑塔纳黑色的宝马,红色的奔驰,灰色的桑塔纳灰色的宝马,白色的奔驰,红色的桑塔纳4 9 克鲁特是德国家喻户晓的“ 明星” 北极熊,北极熊是名副其实的北极霸主,因此,克鲁特是名副其实的北极霸主。以下除哪项外,均与上述论证出现的谬误相似?A 儿童是祖国的花朵,小雅是儿童,因此小雅是祖国的花朵。B 鲁迅的作品不是一天能读完的, 祝福是鲁迅的作品。因 此 祝福不是一天能读完的。C.中国人是不怕困难的,我是中国人,因此,我是不怕困难的。D.康怡花园座落在清水街, 清水街的建筑属于违章建筑。因此, 康怡花园的建筑属于违章建筑E.西班牙是外语,外语

310、是普通高等学校招生的必考科目。因此,西班牙语是普通高校招生的必考科目50. 在本年度篮球联赛中,长江队主教练发现,黄河队五名主力队员之间的上场配置有如下规律:1. 若甲上场,则乙也要上场2 . 只有甲不上场,丙才不上场3 . 要么丙不上场,要么乙和戊中有人不上场4 . 若乙不上场,则以下哪项配置合乎上述规律?A.甲,丙,丁同时上场。B.丙不上场,丁,戊同时匕场。C.甲不上场,丙丁都上场。D.甲,丁都上场,戊不上场。E.甲,丁,戊都不上场51 . 陈先生:未经许可侵入别人的电脑,就好像开偷来的汽车撞伤了人,这些都是犯罪行为。但后者性质更严重,因为它既侵占了有形财产,又造成了人身伤害;而前者只是在

311、虚拟世界中捣乱。林女士:我不同意,例如,非法侵入医院的电脑,有可能扰乱医疗数据,甚至危及病人的生命。因此,非法侵入电脑同样会造成人身伤害。以下哪项最为准确的概括了两人争论的焦点?A.非法侵入别人的电脑和开偷来的汽车是否同样会危及人的生命?B.非法侵入别人的电脑和开偷来的汽车伤人是否都构成犯罪?C.非法侵入别人电脑和开偷来的汽车伤人是否是同样性质的犯罪?D.非法侵入别人电脑的犯罪性质是否和开偷来的汽车伤人一样严重?E.是否只有侵占有形财产才构成犯罪?52 . 小明,小红,小丽,小强,小梅五人去听音乐会。他们五人在同一排且座位相连,其中只有一个座位最靠近走廊,如果小强想坐在最靠近走廊的座位上,小丽

312、想跟小明紧挨着,小红不想跟小丽紧挨着,小梅想跟小丽紧挨着,但不想跟小强或小明紧挨着。以下哪项排序符合上述五人的意愿?A.小明,小梅,小丽,小红,小强B.小强,小红,小明,小丽,小明C.小强,小梅,小红,小丽,小强D.小明,小红,小梅,小丽,小强E.小强,小丽,小梅,小明,小红53. 参加某国际学术研讨会的6 0 名学者中,亚裔学者31人,博士 3 3 人,非亚裔学者中无博士学位的4 人。根据上述陈述,参加此次国际研讨会的亚裔博士有几人?A.1人 B.2人 C.4人 D.7人 E.8人5 4 对某高校本科生的某项调查统计发现:在因成绩优异被推荐免试攻读硕士研究生的文科专业生中,女生占有7 0 %

313、 ,由此可见, 该校本科生专业的女生比男生优秀。A 在该校本科文科专业学生中, 女生占30%以上。B 在该校本科文科专业学生中, 女生占30%以下。C 在该校本科文科专业学生中, 男生占30%以下。D 在该校本科文科专业学生中, 男生占30%以下。E 在该校本科文科专业学生中, 男生占70%以上。5 5 某中药配方有如要求:(1)如果有甲药材,那么也要有乙药材;(2 )如果没有丙药材,那么必须有丁药材;(3)人参和天麻不能都有;( 4)如果没有甲药材而有丙药材,则需要有人参。如果含有天麻,则关于该配方的断定哪项为真?(A ) 含有甲药材(B)含有丙药材(C)没有丙药材(D)没有乙药材和丁药材(

314、E)含有乙药材或丁药材四、写作:第 5657小题,共 65分。其中论证有效性分析30分,论说文35分。请写在答题纸指定位置上。56 . 论证有效性分析:分析下列论证中存在的缺陷和漏洞,选择若干要点,写一篇600字左右的文章, 对该论证的有效性分析进行分析和评述。 ( 论证有效性分析的一般要求是:概念特别是核心概念的界定和使用是否准确并前后一致, 有无各种明显的逻辑错误, 论证的论据是否成立并支持结论,结论成立的条件是否充分等等。 )美国学者弗里德曼的 世界是平的一书认为,全球化对当代人类社会的思想、经济、政治和文化等领域产生了深刻影响。全球化抹去了各国的疆界,是世界从立体变成了平面,也就是说,

315、世界各国之间的社会发展差距正在日益缩小。“ 世界是平的” 这一观点,是基于近几卜年信息传播技术迅猛发展的状况而提出的,互联网的普及、软件的创新使海量信息迅速扩散到世界各地,由于世界是平的,穷国可以和富国一 样在同一平台上接受同样的最新信息。 这样就大大促进了穷国的经济发展, 而改善了它们的国际地位。事实也是如此,所谓“ 金砖四国” 国际声望的上升,无不得益于他们的经济成就,无不得益于互联网技术的发展。特别是中国经济的起飞,中国在世界上的崛起,无疑也依靠了互联网技术的普及,同时也可作为“ 世界是平的” 这观点的有力佐证。毋庸置疑,信息传播技术革命还远未结束,互联网技术将会有更大发展,人类社会将有

316、更惊人的变化,可以预言,由于信息技术的迅猛发展,世界的经济格局与政治格局将会发生巨大的变化,世界最不发达的国家和最发达的国家之间再也不会让人有天壤之别的感觉,非洲大陆将会成为另一个北美。同样也可以预言,由于中国信息技术发展迅猛,中国和世界一样,也会从立体变为平面,中国东西部之间的经济鸿沟将被填平,中国西部的崛起指“可待。57 . 论说文:根据下述材料,写一篇700字左右的论说文,题目自拟。一个真正的学者, 其崇高使命是追求真理。 学者个人的名利乃至生命与之相比都微不足道, 但因为其献身于真理就会变得无限伟大。 一些著名大学的校训中都含有追求真理的内容。然而, 近年学术界的一些状况与追求真理这一

317、使命相去甚远, 部分学者的功利化倾向越来越严重,抄袭剽窃、学术造假、自我炒作、沽名钓誉等等现象时有所闻。1-5: DCCAD 6-10: EBBBD 11-15: ACBBE 16-20: AACDD 21-25: EDACB26-30: DACED 31-35: BDEDB 36-40: ACCAA 41-45: DBEEC 46-50: ADADC51-55: DBECE写作部分答案请参照华章教师授课内容( 六)2010年MBA英语真题及答案Section I Use of EnglishDirections:Read the following text. Choose the best

318、 word(s) for each numbered blank and mark A, B,C or D on ANSWER SHEET 1. (10 points)The outbreak of swine flu that was first detected in Mexico was declared a globalepidemic on June 11, 2009. It is the first worldwide epidemic 1 _ by the World HealthOrganization in 41 years.The heightened alert _2_

319、an emergency meeting with flu experts in Geneva thatassembled after a sharp rise in cases in Australia, and rising 3 in Britain, Japan, Chile andelsewhere.But the epidemic is 4 in severity, according to Margaret Chan, the organizationsdirector general, 5 the overwhelming majority of patients experie

320、ncing only mildsymptoms and a full recovery, often in the _6_ of any medical treatment.The outbreak came to global _7_ in late April 2009, when Mexican authorities notedan unusually large number of hospitalizations and deaths 8 healthy adults.As much of Mexico City shut down at the height of a panic

321、, cases began to _9_ in NewYork City, the south western United States and around the world.In the United States, new cases seemed to fade 10_ warmer weather arrived. But inlate September 2009, officials reported there was 11_ flu activity in almost every state andthat virtually all the 12_ tested ar

322、e the new swine flu, also known as (A) H1N1m, notseasonal flu. In the U.S., it has 13_ more than one million people, and caused more than600 deaths and more than 6,000 hospitalizations.Federal health officials 14 Tamiflu for children from the national stockpile and began15 orders from the states for

323、 the new swine flu vaccine. The new vaccine, which isdifferent from the annual flu vaccine, is 16 ahead of expectations. More than three milliondoses were to be made available in early October 2009, though most of those 17_ doseswere of the FluMist nasal spray type, which is not 18 for pregnant wome

324、n, people over50 or those with breathing difficulties, heart disease or several other 19 . But it was stillpossible to vaccinate people in other high-risk groups: health care workers. People 20infants and healthy young people.1.A. criticizedB. appointedC. commentedD. designated2.A. proceededB. activ

325、atedC. followedD. prompted3.A. digitsB. numbersC. amountsD. sums4.A. moderateB. normalC. unusualD. extreme5.A. withB. inC. fromD. by6.A. progressB. absenceC. presenceD. favor7.A. realityB. phenomenonC. conceptD. notice8.A. overB. forC. amongD. to9.A. stay upB. crop upC. fill upD.cover up10. A. asB.

326、ifC. unlessD. until11. A. excessiveB. enormousC. significantD. magnificent12. A. categoriesB. examplesC. patternsD. samples13. A. impartedB. immersedC. injectedD. infected14. A. releasedB. relayedC. relievedD. remained15. A. placingB. deliveringC. takingD. giving16. A. feasibleB. availableC. reliabl

327、eD. applicable17. A. prevalentB. principalC. innovativeD. initial18. A. presentedB. restrictedC. recommendedD. introduced19. A. problemsB. issuesC. agoniesD. sufferings20. A. involved inB. caring forC. concerned withD. warding offSection II Reading ComprehensionPart ADirections:Read the following fo

328、ur texts. Answer the questions below each text by choosing A, B, Cor D. Mark your answers on ANSWER SHEET 1. (40 points)Text 1The longest bull run in a century of art-market history ended on a dramatic note with asale of 56 works by Damien Hirst, Beautiful Inside My Heart Forever, at Sothebys inLond

329、on on September 15th 2008. All but two pieces sold, fetching more than 70m, arecord for a sale by a single artist. It was a last victory. As the auctioneer called out bids, inNew York one of the oldest on Wall Street, Lehman Brothers. filed for bankruptcy.The world art market had already been losing

330、 momentum for a while after risingbewilderingly since 2003. At its peak in 2007 it was worth some $65 billion, reckons ClareMcAndrew, founder of Arts Economics, a research firm-double the figure five years earlier.Since then in may have come down to $50 billion. But the market generates interest far

331、beyond its beyond its size because it brings together great wealth, enormous egos, greed,passion and controversy in a way matched by few other industries.In the weeks and months that followed Mr. Hirsts sale, spending of any sort becamedeeply unfashionable. In the art word that meant collectors stay

332、ed away from galleriesand salerooms. Sales of contemporary art fell by two-thirds, and in the most overheatedsector, they were down by nearly 90%in the year to November 2OO8.Within weeks theworlds two biggest auction houses, Sothebys and Christies, had to pay out nearly $200min guarantees to clients

333、 who had placed works for sale with them.What makes this slump different form the last, he says, is that there are still buyers inthe market. Almost everyone who was interviewed for this special report said that thebiggest problem at the moment is not a lack of demand but a lack of good work to sell

334、. Thethree Ds -death, death, debt and divorce - still deliver works of art to the market. Butanyone who does not have to sell is keeping away, walling for confidence to return.21. In the first paragraph, Damien Hirsts sale was referred to as a last victory1 becauseA. the art market had witnessed a s

335、uccession of victoriesB. the auctioneer finally got the two pieces at the highest bidsC. Beautiful Inside My Heart Forever won over all masterpiecesD. it was successfully made just before the world financial crisis22. By saying spending of any sort became deeply unfashionable* (Lin1-2, Para.3), thea

336、uthor suggests t hat .A. collectors were no longer actively involved in art-market auctionsB. people stopped every kind of spending and stayed away form galleriesC. art collection as a fashion had lost its appeal to a great extentD. works of art in general had gone out of fashion so they were not wo

337、rth buying23. Which of the following statements NOT true?A. Sales of contemporary art fell dramatically from 2007to 2008.B. The art market surpassed many other industries in momentum.C. The art market generally went downward in various ways.D. Some art dealers were awaiting better chances to come.24

338、. The three Ds mentioned in the last paragraph are.A. auction houses favoritesB. contemporary trendsC. factors promoting artwork circulationD. styles representing Impressionists25. The most appropriate title for this text could be.A. Fluctuation of Art PricesB. Up-to-date Art AuctionsC. Art Market i

339、n DeclineD. Shifted Interest in ArtsText 2I was addressing a small gathering in a suburban Virginia living room-a womensgroup that had-invited men to join them. Throughout the evening, one man had beenparticularly talkative, frequently offering ideas and anecdotes, while his wife sat silentlybeside

340、him on the couch. Toward the end of the evening, I commented that womenfrequently complain that their husbands dont talk to them. This man quickly nodded inagreement. He gestured toward his wife and said, shes the talker in our family.* Theroom burst into laughter the man looked puzzled and hurt. It

341、s true,* he explained.* WhenI come home from work I have nothing to say. If she didn*t keep the conversation going,we*d spend the whole evening in silence.This episode crystallizes, the irony that although American men tend to talk more thanwomen in public situations, they often talk less at home. A

342、nd this pattern is wreakinghavoc with marriage.The pattern was observed by political scientist Andrew Hacker in the late 1970s.Sociologist Catherine Kohler Riessman reports in her new book Divorce Talk that most ofthe women she interviewed- but only a few of the men - gave lack of nearly 50 percent,

343、that amounts to millions of cases in the United States every year - a virtual epidemic offailed conversation.In my own research, complaints form women about their husbands most oftenfocused not on tangible inequities such as having given up the chance for a career toaccompany a husband to his , or d

344、oing far more than their share of daily life-support worklike cleaning, cooking and social arrangements. Instead, they focused, as Hackerobserved years before, that most wives want their husbands to be, first and foremost,conversational partners, but few husbands share this expectation of their wive

345、s.In short, the image that best represents the current crisis is the stereotypical cartoonscene of a man sitting at the breakfast table with a newspaper help up in front of his face,while a woman glares at the back of it, wanting to talk.26. What in most wives main expectation of their husbands?A. T

346、alking to them.B. Trusting them.C. Supporting them.D. Sharing housework.27. Judging from the context. The phrase wreaking havoc(Line 3, para.2) most probablymeans .A. generating motivationB. exerting influenceC. causing damageD. creating pressure28. All of the following are true E X CE P T .A. men t

347、end to talk more in public than womenB. nearly 50 percent of recent divorces are caused by failed conversationC. women attach much importance to communication between couplesD. a female tends to be more talkative at home than her spouse29. Which of the following can best summarize the main idea of t

348、his text?A. The moral decaying deserves more research by sociologists.B. Marriage break-up stems from sex inequalities.C. Husband and wife have different expectations from their marriage.D. Conversational patterns between man and wife are different.30. In the following part immediately after this te

349、xt, the author will most probably focus onA. a vivid account of the new book Divorce TalkB. a detailed description of the stereotypical cartoonC. other possible reasons for a high divorce rate in the U.SD. a brief introduction to the political scientist Andrew HackerText3Over the past decade, many c

350、ompanies had perfected the art of creating automaticbehaviors-habits-among customers. These habits have helped companies earn billions ofdollars when customers eat snacks or wipe counters almost without thinking, often inresponse to a carefully designed set of daily cues.There are fundamental public

351、 health problems, like dirty hands instead of a soaphabit, that remain killers only because we cant figure out how to change peoples habits/*said Dr. Curtis, the director of the Hygiene Center at the London School of Hygiene&Tropical Medicine. uWe wanted to learn from private industry how to create

352、new behaviorsthat happen automatically.MThe companies that Dr. Curtis turned to - Procter & Gamble, Colgate-Palmolive andUnilever - had invested hundreds of millions of dollars finding the subtle cues inconsumers* lives that corporations could use to introduce new routines.If you look hard enough, y

353、oull find that many of the products we use every day. Today,because of shrewd advertising and public health campaigns, many Americans habituallygive their pearly whites a cavity-preventing scrub twice a day, often with Colgate, Crest orone of the other brands.A few decades ago, many people didn*t dr

354、ink water outside of a meal. Then beveragecompanies started bottling the production of far-off springs, and now office workersunthinkingly sip bottled water all day long. Chewing gum, once bought primarily byadolescent boys, is now featured in commercials as a breath freshener and teeth cleanserfor

355、use after a meal. Skin moisturizers are advertised as part of morning beauty rituals,slipped in between hair brushing and putting on makeup.“Our products succeed when they become part of daily or weekly patterns,saidCarol Berning, a consumer psychologist who recently retired from Procter & Gamble, t

356、hecompany that sold $76 billion of Tide, Crest and other products last year. “Creatingpositive habits is a huge part of improving our consumers, lives, and it,s essential tomaking new products commercially viable.”Though experiments and observation, social scientists like Dr. Berning have learnedtha

357、t there is power in tying certain behaviors to habitual cues through ruthless advertising.As this new science of habit has emerged, controversies have erupted when the tacticshave been used to sell questionable beauty creams or unhealthy foods.31. According to Dr. Curtis, habits like hand washing wi

358、th soap.A. should be further cultivatedB. should be changed graduallyC. are deeply rooted in historyD. are basically private concerns32. Bottled water, chewing gum and skin moisturizers are mentioned in Paragraph 5 so asto.A. reveal their impact on peoples habitsB. show the urgent need of daily nece

359、ssitiesC. indicate their effect on peoples buying powerD. manifest the significant role of good habits33. Which of the following does NOT belong to products that help create peoples habits?A. TideB. CrestC. ColgateD. Unilever34. From the text we know that some of consumers, habits are developed due

360、to.A. perfected art of productsB. automatic behavior creationC. commercial promotionsD. scientific experiments35. The authors attitude toward the influence of advertisement on peoples habits isA. indifferentB. negativeC. positiveD. biasedText4Many Americans regard the jury system as a concrete expre

361、ssion of crucialdemocratic values, including the principles that all citizens who meet minimalqualifications of age and literacy are equally competent to serve on juries; that jurorsshould be selected randomly from a representative cross section of the community; thatno citizen should be defined the

362、 right to serve on a jury on account of race, religion, sex ornational origin; that defendants are entitled to trial by their peers; and that verdicts shouldrepresent the conscience of the community and not just the letter of law. The jury is alsosaid to be the best surviving example of direct rathe

363、r than representative democracy. In adirect democracy, citizens take turns governing themselves, rather than electingrepresentatives to govern forthem.But as recently as in 1968, jury selection procedures conflicted with these democraticideals. In some states, for example, jury duty was limited to p

364、ersons of supposedlysuperior intelligence, education, and moral character. Although the Supreme Court of theUnited States had prohibited intentional racial discrimination in jury selection as early asthe 1880 case of Strauder v. West Virginia, the practice of selecting so-called elite orblue-ribbon

365、juries provided a convenient way around this and other antidiscriminationlaws.The system also failed to regularly include women on juries until the mid-20th century.Although women first served on state juries in Utah in 1989, it was not until the 1940s thata majority of states made women eligible fo

366、r jury duty. Even then several statesautomatically exempted women from jury duty unless they personally asked to have theirnames included on the jury list. This practice was justified by the claim that women wereneeded at home, and it kept juries unrepresentative of women through the 1960s.In 1968,

367、the Congress of the United States passed the Jury Selection and Service Act,ushering in a new era of democratic reforms for the jury. This law abolished specialeducational requirements for federal jurors and required them to be selected at randomfrom a cross section of the entire community. In the l

368、andmark 1975 decision Taylor v.Louisiana, the Supreme Court extended the requirement that juries be representative ofall parts of the community to the state level. The Taylor decision also declared sexdiscrimination in jury selection to be unconstitutional and ordered states to use the sameprocedure

369、s for selecting male and female jurors.36. From the principles of the US jury system, we learn thatA. both literate and illiterate people can serve on juries.B. defendants are immune from trial by their peersC. no age limit should be imposed for jury serviceD. judgment should consider the opinion of

370、 the public37. The practice of selecting so-called elite jurors prior to 1968 showedA. the inadequacy of antidiscrimination lawsB. the prevalent discrimination against certain racesC. the conflicting ideals in jury selection proceduresD. the arrogance common among the Supreme Court judges38. Even in

371、 the 1960s, women were seldom on the jury list in some statesb e c a u s e .A. they were automatically banned by stated lawsB. they fell far short of the required qualificationsC. they were supposed to perform domestic dutiesD. they tended to evade public engagement39. After the Jury Selection and S

372、ervice Act was p a s s e d , .A. sex discrimination in jury selection was unconstitutional and had to be abolishedB. educational requirements became less rigid in the selection of federal jurorsC. jurors at the state level ought to be representative of the entire communityD. states ought to conform

373、to the federal court in reforming the jury system40. In discussing the US jury system, the text centers on.A. its nature and problemsB. its characteristics and traditionC. its problems and their solutionsD. its tradition and developmentPart BDirections:Read the following text and decide whether each

374、 of the statements is true or false.Choose T if the statement is true or F if the statement is not true. Mark your answers onANSWER SHEET 1.(10 points)Copying Birds May Save Aircraft FuelBoth Boeing and Airbus have trumpeted the efficiency of their newest aircraft, the 787and A350 respectively. Thei

375、r clever designs and Stanford University, led by Hand Kroo,has suggested that airlines could take a more naturalistic approach to cutting jet-fuel use,and it would not require them to buy new aircraft.The answer, says Dr Kroo, lies with birds. Since 1914, scientists have known thatbirds flying in fo

376、rmation-a V-shape- expend less energy. The air flowing over a birds wingscurls upwards behind the wingtips, a phenomenon known as upwash. Other birds flying inthe upwash experience reduced drag, and spend less energy propelling themselves.Peter Lissaman, an aeronautics expect who was formerly at Cal

377、tech and University ofSouthern California, has suggested that a formation of 25 birds might enjoy a rangeincrease of 71%.When applied to aircraft, the principles are not substantially different. Dr Kroo and histeam modeled what would happen if three passenger jets departing from Los Angeles,San Fran

378、cisco and Las Vegas were to assemble over Utah, assume an invertedV-formation, occasionally change places so all could have a turn in the most favourablepositions, and proceed to London. They found that the aircraft consumed as much as 15%less fuel (coupled with a reduction in carbon-dioxide output)

379、. Nitrogen-oxide emissionsduring the cruising portions of the flight fell by around a quarter.There are, of course, knots to be worked out. One consideration is safety, or at leastthe perception of it. Would passengers feel comfortable in companion? Dr Kroo points outthat the aircraft could be separ

380、ated by several nautical miles, and would not be in theintimate groupings favoured by display teams like the Red Arrows. A passenger peeringout of the window might not even see the other planes. Whether the separation distancesinvolved would satisfy air-traffic-control regulations is another matter,

381、 although a workinggroup at the International Civil Aviation Organisation has included the possibility offormation flying in a blueprint for new operational guidelines.It remains to be seen how weather conditions affect the air flows that make formationflight more efficient. In zones of increased tu

382、rbulence, the planes5 wakes will decay morequickly and the effect will diminish. Dr Kroo says this is one of the areas his team willinvestigate further. It might also be hard for airlines to co-ordinate the departure times anddestinations of passenger aircraft in a way that would allow them to gain

383、from formationflight. Cargo aircraft, in contrast, might be easier to reschedule, as might routine militaryflights.As it happens, Americas armed forces are on the case already. Earlier this year thecountrys Defence Advanced Research Projects Agency announced plans to pay Boeingto investigate formati

384、on flight, though the programme has yet to begin. There are reportsthat some military aircraft flew in formation when they were low on fuel during the SecondWorld War, but Dr Lissaman says they are unsubstantiated. uMy father was an RAF pilotand my cousin the skipper of a Lancaster lost over Berllin

385、/ he adds. So he should know.41. Findings of the Stanford University researchers will promote the sales of new Boeingand Airbus aircraft.42. The upwash experience may save propelling energy as well as reducing resistance.43. Formation flight is more comfortable because passengers can not see the oth

386、erplanes.44. The role that weather plays in formation flight has not yet been clearly defined.45. It has been documented that during World War II, Americas armed forces once triedformation flight to save fuel.Section III Translation46. Directions:In this section there is a text in English. Translate

387、 it into Chinese. Write your translation onANSWER SHEET 2. (15 points)“Sustainability has become a popular word these days, but to Ted Ning, the conceptwill always have personal meaning. Having endured a painful period of unsustainability inhis own life made it clear to him that sustainability-orien

388、ted values must be expressedthrough everyday action and choice.Ning recalls spending a confusing year in the late 1990s selling insurance. Hed beenthrough the dot-com boom and burst and, desperate for a job, signed on with a Boulderagency.It didnt go well. ult was a really bad move because thafs not

389、 my passion71 says Ning,whose dilemma about the job translated, predictably, into a lack of sales. ul was miserable.I had so much anxiety that I would wake up in the middle of the night and stare at theceiling. I had no money and needed the job. Everyone said, *Just wait, youll turn thecorner, give

390、it some time”Section IV WritingPart A47. Directions:You have just come back from the U.S. as a member of a Sino-American culturalexchange program. Write a letter to your American colleague to1) express your thanks for his/her warm reception;2) welcome him/her to visit China in due course.You should

391、write about 100 words on ANSWER SHEET 2.Do not sign your own name at the end of the letter. Use “Zhang Wein instead.Do not write your address.(10 points)Part B48.Directions:In this section, you are asked to write an essay based on the following chart. In yourwriting, you should1) interpret the chart

392、 and2) give your comments.You should write at least 150 words.Write your essay on ANSWER SHEET 2.(15 points)Mobile-phone subscriptions(2000-2008)I I Developing countriesDeveloped countriessubscription number客观题部分答案- 华章名师顾越老师版1-5: DDBAA 6-10: BDCBA 11-15: ADDAA 16-20: BDCDB 21-25: DCCCC26-30: ACBCB 3

393、1-35: ADDCB 36-40: DCCBC 41-45: FTFTF46: “ 可持续性” 已经成为当前个热门词汇,但对于泰德. 宁来说,这个词自有另潘含义。在经历了人生中一段痛苦的、难以为续的阶段后,泰德. 宁清楚地认识到,可持续发展导向的价值必须通过日常的每个行动和选择来实现。宁回忆起了在上世纪90年代末期有一份卖保险的那段混沌经历。在经历了互联网的繁荣和大发展后,宁迫切地需要一份工作,于是,他同博德的一家中介机构签了约。事情进展的很不顺利,宁说:“ 那真是错误的一步,因为那根本不是我的兴趣所在” 。可以想见,在这种困扰的背后则是低迷的销售业绩。“ 我当时真是可怜。我如此地焦虑,以至于总是在半夜醒来,然后盯着天花板。我没有钱并且需要这份工作,每个人都和我说,再等等,总会有转机,再多点时间!”

展开阅读全文
相关资源
正为您匹配相似的精品文档
相关搜索

最新文档


当前位置:首页 > 建筑/环境 > 施工组织

电脑版 |金锄头文库版权所有
经营许可证:蜀ICP备13022795号 | 川公网安备 51140202000112号